*NURSING > UWorld > UWorld Medical-Surgical Nursing Test Questions and Answers (Verified 2021) (All)

UWorld Medical-Surgical Nursing Test Questions and Answers (Verified 2021)

Document Content and Description Below

UWorld Medical-Surgical Nursing Test Test Id: 52084841 Question Id: 32803 (729561) The nurse is providing care for a client with Alzheimer disease who often becomes angry and agitated 20 ... minutes or more after eating. The client accuses the nurse of not providing food, saying, "I'm hungry. You didn't feed me." The nurse should take which action? Unordered Options Ordered Response 1. Give the client gentle reminders that the client has already eaten 2. Say that the client can have a snack in a couple of hours 3. Serve the client half of the meal initially and offer the other half later 4. Take a picture of the client having a meal and show it when the client becomes upset Explanation Most clients with Alzheimer disease experience eating and nutritional problems throughout the course of their disease. During the earlier stages, it is common for clients to forget that they have eaten recently. The best approach is for caregivers to give clients something to eat when they say they are hungry. Smaller meals throughout the day, along with low-calorie snacks, are effective strategies for clients who forget that they have eaten. (Option 1) Reality orientation has been recommended in the past as a way to deal with confusion (eg, dementia, Alzheimer disease), but research has shown that it may cause anxiety and distress. Validation therapy is a newer and more therapeutic approach that validates and accepts the client's reality. (Option 2) Offering to provide a snack later does not address the client's stated need to eat now. Delay in giving food will only further increase the client's anger and frustration. (Option 4) Showing a picture of the client having a meal is confrontational and will have no meaning to the client. Educational objective: Clients with Alzheimer disease experience eating and nutritional problems throughout the course of the disease. During the earlier stages, it is common for them to forget that they have eaten recently. The best approach is for caregivers to give clients something to eat when they say they are hungry. Test Id: 52050973 Question Id: 33389 (729561) The nurse cares for a client who had an abdominal aortic aneurysm repair 6 hours ago. Which assessment would require immediate follow-up? Unordered Options Ordered Response 1. Abdomen is soft, nondistended, and tender to touch 2. Blood pressure is 96/66 mm Hg and apical pulse is 112/min 3. Client rates pain as 4 on a scale of 0-10 4. Green bile is draining from the nasogastric tube Explanation Abdominal aortic aneurysms are surgically repaired when they measure about 6 cm or are causing symptoms. Repair can be done via femoral percutaneous placement of a stent graft (endovascular aneurysm repair) or via an open surgical incision of the aneurysm with synthetic graft placement. The client must be monitored postoperatively for graft leakage and hemodynamic stability. Adequate blood pressure is necessary to maintain graft patency, and prolonged hypotension can lead to the formation of graft thrombosis. Signs of graft leakage include a decreasing blood pressure and increasing pulse rate. (Option 1) Following surgery, the client will experience abdominal tenderness. The abdomen should remain soft and nondistended. A rigid, distended abdomen would indicate possible blood (graft leakage) in the cavity. (Option 3) Pain is an expected finding following abdominal surgery. However, increasing pain that is not relieved by medication can indicate possible graft leakage and should be investigated. (Option 4) During abdominal surgeries, it is customary to insert a nasogastric tube that is left in place during the immediate postoperative period. Green bile-colored drainage would be expected. Bloody drainage would cause concern. Educational objective: Following repair of an abdominal aortic aneurysm, hemodynamic stability is a priority. Prolonged hypotension can lead to graft thrombosis. A falling blood pressure and rising pulse rate can also signify graft leakage. Test Id: 52050973 Question Id: 30627 (729561) The nurse has just administered a dose of 0.5 mg atropine to a client with a heart rate of 48/min and blood pressure of 90/62 mmHg. Which rhythm strip would indicate that the medication achieved the desired outcome? Unorder ed Options Ordered Response . . . Explanation Atropine is given to the client experiencing symptomatic bradycardia. In symptomatic bradycardia, the heart rate is <60/min and is inadequate for the client's condition, causing symptoms such as hypotension, chest pain, or syncope. Atropine acts to increase the heart rate by inhibiting the action of the vagus nerve (parasympatholytic effect). A normal sinus rhythm and reversal of clinical symptoms indicate that the medication has had the desired effect. (Option 1) A continuation of sinus bradycardia would not indicate that the atropine had been effective. (Option 3) Sinus tachycardia would be an undesirable effect of atropine as the heart rate would be >100/min. (Option 4) The client with first-degree atrioventricular block may have a normal heart rate, but the atrioventricular conduction time is prolonged. Educational objective: Atropine is given to the client with symptomatic bradycardia. The desired outcome would be an increase in heart rate, evidence of normal sinus rhythm on the cardiac monitor, and reversal of any clinical symptoms associated with the bradycardia. Test Id: 52050973 Question Id: 32111 (729561) The nurse provides post-procedure teaching for a female client who had a cystoscopy as an outpatient. Which client statement indicates the need for additional instruction? Unordered Options Ordered Response 1. "I can expect pink-tinged urine for at least 24 hours." 2. "I can take a warm bath and acetaminophen if I have discomfort or bladder spasms." 3. "I should expect frequency and burning when I urinate." 4. "I should expect to see blood clots in my urine for up to 24 hours." Explanation A cystoscopy is a procedure that uses a flexible fiber-optic scope inserted through the urethra into the urinary bladder with the client in the lithotomy position. Complications associated with cystoscopy include urinary retention, hemorrhage, and infection. Therefore, clients are instructed to notify the health care provider (HCP) immediately if they have bright red blood when urinating, blood clots, inability to urinate, fever >100.4 F (38 C) and chills, or abdominal pain unrelieved by analgesia. These conditions necessitate evaluation by the HCP and may require antibiotic therapy or the insertion of a urinary catheter to irrigate the bladder, remove clots, or drain the bladder (Option 4). (Options 1 and 3) Pink-tinged urine, frequency, and dysuria are expected for up to 48 hours following a cystoscopy. Clients are instructed to increase fluids, drink 4-6 glasses of water daily to help dilute the urine, and avoid alcohol and caffeine for 24-48 hours as these can irritate the bladder. (Option 2) Abdominal discomfort and bladder spasms may occur for up to 48 hours following the procedure. Clients are taught to take a mild analgesic (eg, acetaminophen, ibuprofen) and a warm tub/sitz bath (except with recurrent urinary tract infections) for pain relief. Educational objective: Clients can expect pink-tinged urine, frequency, dysuria, and abdominal discomfort for up to 48 hours after cystoscopy. They are instructed to increase fluid intake, avoid alcohol and caffeine, take a mild analgesic and tub/sitz bath to relieve discomfort, and notify the HCP immediately of inability to void, gross hematuria, blood clots, fever, chills, or severe pain. Test Id: 52050973 Question Id: 31884 (729561) A client with heart failure is started on furosemide. The laboratory results are shown in the exhibit. The nurse is most concerned about which condition? Click on the exhibit button for additional information. Unordered Options Ordered Response 1. Atrial fibrillation 2. Atrial flutter 3. Mobitz II 4. Torsades de pointes Explanation Hypomagnesemia (normal: 1.5-2.5 mEq/L [0.75-1.25 mmol/L]) causes a prolonged QT interval that increases the client's susceptibility to ventricular tachycardia. Torsades de pointes is a type of polymorphic ventricular tachycardia coupled with a prolonged QT interval; it is a lethal cardiac arrhythmia that leads to decreased cardiac output and can develop quickly into ventricular fibrillation. The American Heart Association recommends treatment with IV magnesium sulfate. (Option 1) Characteristics of atrial fibrillation (AF) include an irregularly irregular rhythm and replacement of P waves by fibrillatory waves. Although electrolyte disturbances increase the likelihood of developing AF, clients can have this chronic condition managed with anticoagulation therapy. AF is usually associated with an underlying heart disease and is rarely immediately life-threatening. (Option 2) Atrial flutter is characterized by sawtooth-shaped flutter waves. There is no clinical evidence suggesting that hypomagnesemia leads to atrial flutter, which is associated with underlying heart disease (eg, mitral valve disorders, cardiomyopathy, cor pulmonale). (Option 3) Mobitz II (type II second-degree atrioventricular block) is usually not associated with electrolyte disturbances but is more often associated with conduction system disease or drug toxicity (eg, beta blockers, calcium channel blockers). Educational objective: In a client with hypomagnesemia, it is important to assess the QT interval. The client is most at risk for torsades de pointes, a serious complication that can develop quickly into ventricular fibrillation (lethal arrhythmia). Copyright © UWorld. All rights reserved. Test Id: 52050973 Question Id: 31083 (729561) An elderly client with oxygen-dependent chronic obstructive pulmonary disease is admitted for pneumonia. The client is do not resuscitate, and the nurse is concerned that the client will soon develop respiratory failure as breathing is becoming shallow and the client is looking exhausted. Which is the most appropriate intervention to include in the plan of care? Unordered Options Ordered Response 1. Administer morphine to decrease air hunger 2. Call the health care provider for possible intubation 3. Promote relaxation through music and distraction 4. Titrate oxygen to maintain an oxygen saturation ≥94% Explanation A client with oxygen-dependent chronic obstructive pulmonary disease (COPD) and pneumonia who is showing signs of shallow breathing and fatigue may stop breathing. This client is do not resuscitate (DNR), which makes comfort the goal of care. Interventions should be used to promote comfort, such as relaxation, music, and distraction. Anxiety can be alleviated by being present and talking calmly to the client. Interventions that may compromise breathing and hasten death should be avoided. (Option 1) Morphine is not appropriate in this client with respiratory failure as it may cause respiratory depression, hastening death. (Option 2) Intubation is not appropriate in this client with respiratory failure who is DNR. However, calling the health care provider is appropriate as the client can be offered noninvasive positive pressure ventilation (eg, bilevel positive airway pressure). (Option 4) The oxygen saturation goal in a client with COPD is typically 90%-93%. Educational objective: The nurse should integrate the advance directive into the plan of care for the client. In a client who is DNR, this includes a focus on comfort when the clinical situation is declining. Test Id: 52050973 Question Id: 30086 (729561) The nurse is assessing a 70-year-old client with a long history of type 2 diabetes mellitus for sudden, severe nausea, diaphoresis, dizziness, and fatigue in the emergency department. Which hospital protocol would be the most appropriate to follow initially? Unordered Options Ordered Response 1. Food poisoning 2. Influenza 3. Myocardial infarction 4. Stroke Explanation Early recognition and treatment of heart attack are critical. Women, the elderly, and clients with a history of diabetes may not have the classic heart attack symptoms of dull chest pain with radiation down the left arm. Instead, they can present with "atypical" symptoms such as nausea, vomiting, belching, indigestion, diaphoresis, dizziness, and fatigue. (Option 1) Taking a careful history and evaluating for any sick contacts would be helpful in identifying food poisoning, but a more important initial step is to assess for a heart attack. (Option 2) A viral infection is a possibility, but fever and myalgia are usually present during an episode of influenza. (Option 4) Early intervention in stroke is also critical, and a neurologic assessment would take place after the acute coronary syndrome algorithm, especially with negative electrocardiography and serum heart enzyme levels. Educational objective: Myocardial infarctions in women, the elderly, and diabetics may have gastrointestinal distress as the main symptom; this needs to be evaluated with the institutional protocol for acute coronary syndrome. Test Id: 52050973 Question Id: 30086 (729561) The nurse is assessing a 70-year-old client with a long history of type 2 diabetes mellitus for sudden, severe nausea, diaphoresis, dizziness, and fatigue in the emergency department. Which hospital protocol would be the most appropriate to follow initially? Unordered Options Ordered Response 1. Food poisoning 2. Influenza 3. Myocardial infarction 4. Stroke Explanation Early recognition and treatment of heart attack are critical. Women, the elderly, and clients with a history of diabetes may not have the classic heart attack symptoms of dull chest pain with radiation down the left arm. Instead, they can present with "atypical" symptoms such as nausea, vomiting, belching, indigestion, diaphoresis, dizziness, and fatigue. (Option 1) Taking a careful history and evaluating for any sick contacts would be helpful in identifying food poisoning, but a more important initial step is to assess for a heart attack. (Option 2) A viral infection is a possibility, but fever and myalgia are usually present during an episode of influenza. (Option 4) Early intervention in stroke is also critical, and a neurologic assessment would take place after the acute coronary syndrome algorithm, especially with negative electrocardiography and serum heart enzyme levels. Educational objective: Myocardial infarctions in women, the elderly, and diabetics may have gastrointestinal distress as the main symptom; this needs to be evaluated with the institutional protocol for acute coronary syndrome. Test Id: 52050973 Question Id: 33789 (729561) The nurse is assessing a client an hour after a left lung lobectomy. The client is awake, alert, and oriented, and reports pain of 6 on a 0-10 scale. Pulse oximetry is 92% on 4 L oxygen via nasal cannula. The chest tube is set to continuous water seal suction at -20 cm H2O. The collection chamber has accumulated 320 mL of frank red drainage in the last hour. What is the priority nursing action? Unordered Options Ordered Response 1. Clamp the chest tube immediately 2. Increase oxygen to 6 L via nasal cannula 3. Medicate client for pain and document the findings 4. Notify the health care provider immediately Explanation Following lung surgery, a chest tube is inserted into the pleural space to create a negative vacuum to re-inflate the lung and prevent air from re-entering the space. A client with a chest tube should be assessed for signs of air/fluid in the chest (eg, diminished breath sounds), excessive drainage (>100 mL/hr), pain, and infection at the drainage site. The collection chamber should be inspected every hour for the first 8 hours following surgery, then every 8 hours until it is removed. Excess drainage of frank red blood is indicative of hemorrhage and must be managed immediately. The priority action is to contact the health care provider for further management. (Option 1) Clamping the chest tube prevents air or fluid from leaving the pleural space, which may cause a reciprocal tension pneumothorax. The chest tube is clamped only a few hours prior to removal, momentarily to check for an air leak, or if the drainage apparatus needs to be changed. (Option 2) Although a pulse oximetry of 92% is low, this is an expected finding following lung surgery. (Option 3) Pain following surgery is a concern and the client will require medication; however, hemorrhage is the priority. Educational objective: A client with a chest tube should be assessed for signs of air/fluid in the chest (eg, diminished breath sounds), excessive drainage (>100 mL/hr), pain, and infection at the drainage site. Excess drainage of frank red blood is indicative of hemorrhage and must be managed immediately. Test Id: 52050973 Question Id: 33789 (729561) The nurse is assessing a client an hour after a left lung lobectomy. The client is awake, alert, and oriented, and reports pain of 6 on a 0-10 scale. Pulse oximetry is 92% on 4 L oxygen via nasal cannula. The chest tube is set to continuous water seal suction at -20 cm H2O. The collection chamber has accumulated 320 mL of frank red drainage in the last hour. What is the priority nursing action? Unordered Options Ordered Response 1. Clamp the chest tube immediately 2. Increase oxygen to 6 L via nasal cannula 3. Medicate client for pain and document the findings 4. Notify the health care provider immediately Explanation Following lung surgery, a chest tube is inserted into the pleural space to create a negative vacuum to re-inflate the lung and prevent air from re-entering the space. A client with a chest tube should be assessed for signs of air/fluid in the chest (eg, diminished breath sounds), excessive drainage (>100 mL/hr), pain, and infection at the drainage site. The collection chamber should be inspected every hour for the first 8 hours following surgery, then every 8 hours until it is removed. Excess drainage of frank red blood is indicative of hemorrhage and must be managed immediately. The priority action is to contact the health care provider for further management. (Option 1) Clamping the chest tube prevents air or fluid from leaving the pleural space, which may cause a reciprocal tension pneumothorax. The chest tube is clamped only a few hours prior to removal, momentarily to check for an air leak, or if the drainage apparatus needs to be changed. (Option 2) Although a pulse oximetry of 92% is low, this is an expected finding following lung surgery. (Option 3) Pain following surgery is a concern and the client will require medication; however, hemorrhage is the priority. Educational objective: A client with a chest tube should be assessed for signs of air/fluid in the chest (eg, diminished breath sounds), excessive drainage (>100 mL/hr), pain, and infection at the drainage site. Excess drainage of frank red blood is indicative of hemorrhage and must be managed immediately. Test Id: 52050973 Question Id: 30147 (729561) A client with type I diabetes mellitus is brought to the emergency department by his wife. The client has fruity breath with rapid, deep respirations at 36 breaths per minute, reports abdominal pain, and appears weak. The nurse should anticipate implementation of which prescription(s)? Select all that apply. Unordered Options Ordered Response 1. Administer dextrose 50 mg intravenous (IV) push 2. Instruct client to breathe into a paper bag to treat hyperventilation 3. Perform a fingerstick and serum blood glucose test 4. Prepare to administer an IV infusion of regular insulin 5. Start an IV line and administer a bolus of normal saline Explanation The client is exhibiting the cardinal signs and symptoms of diabetic ketoacidosis (DKA). DKA is an acute life-threatening complication, typically of type I diabetes, characterized by hyperglycemia, ketosis, and acidosis. It is caused by an intense deficit of insulin. Glucose cannot be used properly for energy when this deficit occurs and the body begins to break down fat stores, producing ketones, a byproduct of fat metabolism, resulting in metabolic acidosis. The lack of insulin also results in increased production of glucose in the liver, further exacerbating hyperglycemia. Because some of the symptoms of hypoglycemia and DKA overlap, a blood glucose level should be checked to ensure that hyperglycemia is present. Hyperglycemia can cause osmotic diuresis, leading to dehydration. In addition, ketones are excreted in the urine as the body tries to restore its pH balance. Vital electrolytes such as sodium, potassium, chloride, phosphate, and magnesium become depleted during the process. Cardinal signs of dehydration such as poor skin turgor, dry mucous membranes, tachycardia, orthostatic hypotension, weakness, and lethargy can occur. The nurse should start an IV and bolus the client with normal saline or 1/2 normal saline to reverse dehydration. This should occur prior to treating the hyperglycemia with regular insulin IV infusion. Because insulin promotes water, potassium, and glucose entrance into the cell, it can exacerbate vascular dehydration and imbalance of electrolytes, particularly potassium. A potassium level (along with other electrolytes) should also be assessed prior to beginning the prescribed insulin therapy. Other signs associated with DKA include Kussmaul respirations, deep, rapid respirations that have a fruity/acetone smell as the carbon dioxide is exhaled. This compensatory mechanism results in a lowered PaC02 in an attempt to restore the body's normal pH level and should not be reversed (Option 2). (Option 1) IV dextrose is administered during acute hypoglycemic episodes and would worsen DKA. Educational objective: DKA is an acute life-threatening complication, typically of type I diabetes, characterized by hyperglycemia, ketosis, and acidosis. It is caused by an intense deficit of insulin and should be treated first with rehydration (normal saline) and then insulin administration. Copyright © UWorld. All rights reserved Test Id: 52050973 Question Id: 30316 (729561) A client is being discharged today following a partial gastrectomy. Instructions for recuperating at home would include which of the following? Select all that apply. Unordered Options Ordered Response 1. Avoid high fiber foods 2. Avoid intake of fluids with meals 3. Consume low-carbohydrate meals 4. Have small, frequent meals 5. Maintain a sitting-up position after eating Explanation Up to 50% of clients with partial gastrectomy may experience dumping syndrome. Dumping syndrome occurs when gastric contents are emptied too rapidly into the duodenum and cause a fluid shift into the small intestine. This fluid shift results in hypotension, abdominal pain, diarrhea, nausea, vomiting, dizziness, generalized sweating, and tachycardia. The symptoms usually diminish over time. Dietary recommendations are aimed at delaying gastric emptying and include the following: • Small, frequent meals reduce the amount of food in the stomach at any one time. Eat slowly in a relaxed environment. • Avoid meals high in simple carbohydrates, as these may trigger dumping syndrome as the carbohydrates are broken down into simple sugars. Instead, consume meals high in protein, fat, and fiber, as these take longer to digest and will remain in the stomach longer than carbohydrates (Option 1). • Separate fluids from meals. If fluids are taken with meals, stomach contents pass more easily into the jejunum and worsen symptoms. Fluid intake should be only after or between meals, separated from solid intake by at least 30 minutes. • Avoid sitting up after a meal. Gravity increases gastric emptying. Lying down after meals slows down the gastric emptying and is preferred (Option 5). Educational objective: Dumping syndrome is a complication of gastrectomy. Measures that delay gastric emptying can reduce the risk of dumping syndrome. Clients should eat meals low in carbohydrates and high in fiber, proteins, and fats. Fluids should not be taken with meals. Symptoms usually diminish over time. Test Id: 52050973 Question Id: 33795 (729561) The client recently admitted to the assisted living center has impaired vision related to primary open-angle glaucoma. Select the graphic that best illustrates the effects of glaucoma on the client's vision. Unordered Options Ordered Response 1. 2. 3. 4. Explanation Primary open-angle glaucoma (POAG) is an eye condition characterized by an increase in intraocular pressure and gradual loss of peripheral vision (ie, tunnel vision). The signs/symptoms of POAG develop slowly and include painless impairment of peripheral vision with normal central vision, difficulty with vision in dim lighting, increased sensitivity to glare, and halos observed around bright lights. POAG can lead to blindness if left untreated. (Option 1) Retinal detachment is separation of the retina from the underlying epithelium that allows fluid to collect in the space. The signs/symptoms include sudden onset of light flashes, floaters, cloudy vision, or a curtain appearing in the vision. (Option 2) Age-related macular degeneration is a degenerative eye disease that brings about the gradual loss of central vision, leaving peripheral vision intact. (Option 4) A cataract is cloudiness (ie, opacity) of the lens that may occur at birth or more commonly in older adults. The signs/symptoms of a cataract include painless, gradual loss of visual acuity with blurry vision; scattered light on the lens producing glare and halos, which are worse at night; and decreased color perception. Educational objective: Primary open-angle glaucoma is characterized by an increase in intraocular pressure and gradual loss of peripheral vision (ie, tunnel vision). Test Id: 52050973 Question Id: 31223 (729561) A client is brought to the emergency department due to loss of consciousness after binge drinking at a college party and then taking alprazolam. Pulse oximetry shows 87% on room air. Which findings would the nurse expect to assess on an arterial blood gas? Unordered Options Ordered Response 1. Metabolic acidosis and hyperventilation 2. Metabolic alkalosis and hypoventilation 3. Respiratory acidosis and hypoventilation 4. Respiratory alkalosis and hyperventilation Explanation The combination of excessive alcohol ingestion and the benzodiazepine alprazolam (Xanax) causes respiratory depression, which leads to alveolar hypoventilation secondary to carbon dioxide retention, and respiratory acidosis. Therefore, clients should be advised not to take multiple substances that increase the risk of respiratory depression (eg, opioids, benzodiazepines, alcohol, sedating antihistamines). (Option 1) Diarrhea, ketoacidosis, lactic acidosis, and renal failure can cause metabolic acidosis due to loss of bicarbonate or retention of acids; the lungs would compensate by hyperventilating. (Option 2) Vomiting, gastrointestinal suction, and administration of alkali (ie, sodium bicarbonate) are common causes of metabolic alkalosis; the lungs would compensate by hypoventilating. (Option 4) Hypoxia, anxiety, and pain are common causes of respiratory alkalosis, which is due to alveolar hyperventilation (rapid breathing). Educational objective: Over-sedation, sleep apnea, anesthesia, drug overdose, progressive neuromuscular disease, and chronic obstructive pulmonary disease depress the respiratory center; this leads to alveolar hypoventilation, secondary to carbon dioxide retention, and respiratory acidosis. Test Id: 52050973 Question Id: 32324 (729561) The nurse is answering questions at a hospital-sponsored health fair. What actions should the nurse encourage to help prevent contracting the West Nile virus? Select all that apply. Unordered Options Ordered Response 1. Avoid raw, unpeeled fruits or vegetables 2. Limit contact with infected pets 3. Use insect (mosquito) repellent 4. Wash all bedding in hot water 5. Wear long-sleeved, light-colored clothes Explanation West Nile virus is a mosquito-borne disease (encephalitis) that occurs mainly during the summer months, especially during humid weather. Prevention focuses on avoiding mosquitoes and using an insect repellent. Prevention also includes wearing long sleeves, long pants, and light colors and avoiding outdoor activities at dawn and dusk when mosquitoes are most active (Options 3 and 5). (Option 1) Food and water precautions are emphasized for infectious diseases contracted through contaminated water or food, such as hepatitis A or typhoid (enteric) fever. (Option 2) Limiting contact with infected pets is classic advice for avoiding ringworm, a superficial fungal skin infection. (Option 4) Washing bedding in hot water is a classic instruction to help reduce allergies/asthma (eg, commonly from mites) or scabies (a contagious skin infection caused by mites). Educational objective: West Nile virus is transmitted by an infected mosquito bite. Prevention focuses on avoiding mosquitoes and using a mosquito repellent. Prevention also includes keeping arms and legs covered with light-colored clothing and avoiding outdoor activities at dawn and dusk. Test Id: 52050973 Question Id: 30846 (729561) The nurse cares for a client with an exacerbation of inflammatory bowel disease (IBD). The client tells the nurse about being infected with tuberculosis (TB) 10 years ago but never being medicated. Which prescription is of concern and prompts the nurse to notify the health care provider (HCP)? Unordered Options Ordered Response 1. Lansoprazole 2. Metronidazole 3. Prednisone 4. Sulfasalazine Explanation Tuberculosis is an infection caused by the Mycobacterium tuberculosis microorganism. A client with active, primary TB disease has a positive tuberculin skin test (TST), usually feels sick, has symptoms, and can spread the disease to others if not treated with medications. A client with a latent TB infection (LTBI) has a positive TST, negative chest x-ray, is asymptomatic, cannot transmit the disease to others, and can complete a full course of treatment to prevent activation of the disease. Malignancy, immunosuppressant medications, including chemotherapy, and prolonged debilitating disease (eg, HIV), can convert LTBI to active disease. A client with LTBI who begins treatment with a corticosteroid (Prednisone) is at increased risk for conversion to active TB disease. Therefore, the nurse should notify the HCP. (Option 1) Lansoprazole (Prevacid) is a proton pump inhibitor used to treat ulcer disease, erosive esophagitis, and gastroesophageal reflux disease. It does not convert LTBI to active disease. (Option 2) Metronidazole (Flagyl) is an antimicrobial medication used to treat IBD and does not convert LTBI to active disease. (Option 4) Sulfasalazine (Azulfidine) is a gastrointestinal anti-inflammatory medication used to treat IBD and does not convert LTBI to active disease. Educational objective: A client with LTBI has a positive TST, is asymptomatic, and cannot transmit the disease to others. Malignancy, immunosuppressant medications, chemotherapy, and prolonged debilitating disease Test Id: 52050973 Question Id: 31341 (729561) The nurse is monitoring a client who has been on clopidogrel therapy. What assessments are essential? Select all that apply. Unordered Options Ordered Response 1. Assess for bruising 2. Assess for tarry stools 3. Monitor intake and output 4. Monitor liver function tests 5. Monitor platelets Explanation Antiplatelet therapy (eg, aspirin, clopidogrel, prasugrel, ticagrelor) is initiated to prevent platelet aggregation in clients at risk for myocardial infarction, stroke, or other thrombotic events. This therapy increases the risk of bleeding. The client should be assessed for bruising, tarry stools, and other signs of bleeding. In addition, there is a chance that the clopidogrel can cause thrombotic thrombocytopenia purpura (Options 1, 2, and 5). (Option 3) Monitoring intake and output is indicated while the client is on diuretic medications (eg, furosemide, torsemide, bumetanide) but not for antiplatelet agents. (Option 4) Baseline liver enzymes are obtained for clients taking statins (eg, rosuvastatin, atorvastatin) and isoniazid (for tuberculosis), but not for clopidogrel. Educational objective: Any antiplatelet drug (eg, aspirin, clopidogrel, prasugrel, ticagrelor) can pose a risk for serious bleeding. The nurse should teach the client what to assess for potential bleeding (eg, stool, urine, gums) and bruising. Test Id: 52050973 Question Id: 31316 (729561) A client with severe vomiting and diarrhea has a blood pressure of 90/70 mm Hg and pulse of 120/min. IV fluids of 2-liter normal saline were administered. Which parameters indicate that adequate rehydration has occurred? Select all that apply. Unordered Options Ordered Response 1. Capillary refill is less than 3 seconds 2. Pulse pressure is narrowed 3. Systolic blood pressure drops only when standing 4. Urine output is 360 mL in 4 hours 5. Urine specific gravity is 1.020 Explanation This client's initial vital signs show tachycardia and hypotension, which are classic signs of hypovolemia. Normal capillary refill is less than 3 seconds and is an indication of normal hydration and perfusion (Option 1). Obligatory urine output is 30 mL/hr, and this client has 90 mL/hr. Urine output is one of the best indicators of adequate rehydration (Option 4). The urine specific gravity is within a normal range (1.003 to 1.030), which can indicate normal hydration (Option 5). (Option 2) Narrowing pulse pressure (the difference between systolic and diastolic) is a sign of hypovolemic shock and would not indicate adequate rehydration. The client arrived with a narrow pulse pressure already. (Option 3) This is indicative of orthostatic vital signs. When a client stands, the body normally vasoconstricts to maintain the blood pressure from the effects of gravity. If a client is dehydrated, the body has already maximally vasoconstricted, and there is no compensatory mechanism left to adjust to the position change. Educational objective: Signs of adequate hydration are normal urine specific gravity (1.003 to 1.030), adequate volume of urine output (>30 mL/hr), and capillary refill of less than 3 seconds. Pulse pressure narrows in shock, and positive orthostatic vital signs (decreasing systolic blood pressure and rising heart rate) with position change indicate dehydration. Test Id: 52050973 Question Id: 30428 (729561) The nurse is caring for a client after a motor vehicle accident. The client's injuries include 2 fractured ribs and a concussion. The nurse notes which of the following as expected neurological changes for the client with a concussion? Select all that apply. Unordered Options Ordered Response 1. Asymmetrical pupillary constriction 2. Brief loss of consciousness 3. Headache 4. Loss of vision 5. Retrograde amnesia Explanation A concussion is considered a minor traumatic brain injury and results from blunt force or an acceleration/deceleration head injury. Typical signs of concussion include: 1. A brief disruption in level of consciousness 2. Amnesia regarding the event (retrograde amnesia) 3. Headache These clients should be observed closely by family members and not participate in strenuous or athletic activities for 1–2 days. Rest and a light diet are encouraged during this time. (Options 1 and 4) The following manifestations indicate more serious brain injury and are not expected with simple concussion: • Worsening headaches and vomiting (indicate high intracranial pressure) • Sleepiness and/or confusion (indicate high intracranial pressure) • Visual changes • Weakness or numbness of part of the body Educational objective: Expected neurological changes with a concussion include brief loss of consciousness, retrograde amnesia, and headache. These clients should be observed closely by family members and not participate in strenuous or athletic activities for 1–2 days. Test Id: 52050973 Question Id: 30185 (729561) The nurse cares for a group of clients on a medical surgical floor. The client with which condition is at highest risk for developing syndrome of inappropriate antidiuretic hormone (SIADH)? Unordered Options Ordered Response 1. Carpal tunnel syndrome 2. Diabetes mellitus 3. Sciatica 4. Small cell lung cancer Explanation SIADH is an endocrine condition in which too much ADH is produced, causing water retention, increased total body water, and dilutional hyponatremia (low serum sodium). Some cancer cells, particularly those of small cell lung cancer, have the ability to produce and secrete ADH, leading to SIADH. Other causes include central nervous system disorders (eg, stroke, trauma, neurosurgery) and some commonly used medications (eg, desmopressin, carbamazepine). (Options 1 and 3) Carpal tunnel syndrome is a result of aggravated tendons in the wrists causing narrow, pinched nerves. Sciatica is numbness, tingling, or pain caused by an irritation of the sciatic nerve. Both are examples of peripheral nerve disorders. SIADH is more common among clients with central nerve disorders (eg, stroke, neurosurgery). (Option 2) Diabetes mellitus is an endocrine disorder characterized by hyperglycemia and is not commonly associated with SIADH. Educational objective: ADH is sometimes produced and secreted by cancer cells, especially lung cancer cells causing SIADH, a condition in which too much ADH causes water retention, increased total water, and dilutional hyponatremia. Test Id: 52050973 Question Id: 30789 (729561) A nurse on the medical surgical unit has just received report. Which client should be seen first? Unordered Options Ordered Response 1. Client 1 day post femoral-popliteal bypass grafting who has an intravenous (IV) antibiotic due now 2. Client diagnosed with deep venous thrombosis (DVT) yesterday who reports some chest discomfort and cough 3. Client with hypertension and blood pressure of 180/92 mm Hg who reports a headache 4. Client on fall precautions who just called the nurses' station for assistance in using the bathroom immediately Explanation The client with DVT who is experiencing chest discomfort and cough should be seen first. This client is exhibiting possible signs of pulmonary embolism (PE), which can be a life-threatening complication. Signs and symptoms of PE include dyspnea, hypoxemia, tachypnea, cough, chest pain, hemoptysis, tachycardia, syncope, and hemodynamic instability. The nurse should elevate the head of the bed, administer oxygen, and assess the client. The health care provider should be notified of these findings. (Option 1) The administration of an IV antibiotic is important but should be done after the nurse has assessed the client with DVT. (Option 3) This client is hypertensive and most likely has a headache due to the high blood pressure. The nurse should assess this client after the client with DVT and administer any antihypertensives needed. (Option 4) This client can be delegated to unlicensed assistive personnel who can go to the room immediately. Educational objective: The nurse should prioritize the assessment of any client with DVT who is experiencing respiratory signs and symptoms and/or chest pain due to potential development of PE. Test Id: 52036792 Question Id: 30603 (729561) A client with chronic kidney disease has a large pleural effusion. What findings characteristic of a pleural effusion does the nurse expect to assess? Select all that apply. Unordered Options Ordered Response 1. Bronchial breath sounds 2. Decreased fremitus 3. Diminished lung sounds 4. Hyperresonance on percussion 5. Wheezing Explanation A pleural effusion is an abnormal collection of fluid (>15-20 mL) in the pleural space between the parietal and visceral pleurae that prevents the lung from expanding fully. This results in decreased lung volume, atelectasis, and ineffective gas exchange. Clients commonly have dyspnea on exertion and non-productive cough. Examination shows diminished breath sounds, dullness to percussion, and decreased tactile fremitus. If the effusion is large, the trachea (mediastinum) is deviated to the opposite side. Palpable vibration felt on the chest wall is known as fremitus. Sound travels faster in solids (consolidation) than in an aerated lung, resulting in increased fremitus in pneumonia. The presence of egophony, bronchophony, or whispered pectoriloquy also suggests a consolidative process. Fluid or air outside the lung interrupts the transmission of sound, resulting in decreased fremitus in pleural effusion and pneumothorax (Option 2). (Option 1) Bronchial breath sounds are normally present over the trachea, and vesicular breath sounds are present over the lung parenchyma. However, in consolidation (lobar pneumonia), bronchial breath sounds are present over the lung parenchyma due to over-transmission of sound. Over-transmission is due to airway patency leading to the affected area. Breath sounds are diminished or absent over a pleural effusion or pneumothorax. (Option 4) Percussion shows hyperresonance in clients with pneumothorax and dullness in those with pleural effusions or pneumonia. (Option 5) Wheezing indicates an obstructive process (eg, asthma, chronic obstructive pulmonary disease) and is not typical in pleural effusion. Educational objective: A pleural effusion is an abnormal collection of fluid (>15-20 mL) in the pleural space between the parietal and visceral pleurae that prevents the lung from expanding fully, resulting in decreased lung volume, atelectasis, and ineffective gas exchange. Examination shows diminished breath sounds, dullness to percussion, and decreased tactile fremitus. Copyright © UWorld. All rights reserved. Test Id: 52036792 Question Id: 30639 (729561) A client with atrial fibrillation is being discharged home after being stabilized with medications, including digoxin. Which of the following statements regarding digoxin toxicity indicates that further teaching is needed? Unordered Options Ordered Response 1. "I must visit my health care provider (HCP) to check my drug levels." 2. "I should report to my HCP if I develop nausea and vomiting." 3. "I should tell my HCP if I feel my heart skip a beat." 4. "I will need to increase my potassium intake." Explanation Drug toxicity is common with digoxin due to its narrow therapeutic range. Many contributing factors (eg, hypokalemia) can cause toxicity. However, in the absence of other factors, potassium does not need to be increased just because a client is on digoxin. If the client also takes some other potassium-depleting medications, such as diuretics, potassium supplements may be needed. Signs and symptoms of digoxin toxicity include the following: 1. Gastrointestinal symptoms (eg, anorexia, nausea, vomiting, abdominal pain) are frequently the earliest symptoms (Option 2) 2. Neurologic manifestations (eg, lethargy, fatigue, weakness, confusion) 3. Visual symptoms are characteristic and include alterations in color vision, scotomas, or blindness 4. Cardiac arrhythmias – most dangerous (Option 1) Drug levels are frequently monitored until a steady state is achieved and when changes are expected, such as in clients with chronic kidney disease and electrolyte disturbances (eg, hypokalemia, hypomagnesemia). (Option 3) Digoxin toxicity can result in bradycardia and heart block. Clients are instructed to check their pulse and report to the HCP if it is low or has skipped beats. Educational objective: Drug toxicity is common with digoxin due to its narrow therapeutic range. Drug levels are frequently monitored. Nonspecific gastrointestinal symptoms similar to gastroenteritis are common and can lead to serious cardiac arrhythmias if not recognized Test Id: 52036792 Question Id: 30356 (729561) A client is admitted to the emergency department after a fall with dizziness and light- headedness. Blood pressure is 88/62 mm Hg, and the cardiac monitor displays the rhythm in the exhibit. The nurse recognizes it as which rhythm? Click on the exhibit button for additional information. Unordered Options Ordered Response 1. Complete heart block 2. 1st-degree heart block 3. Sinus bradycardia 4. Sinus rhythm Explanation Sinus bradycardia (SB) has the same conduction pathway as sinus rhythm, but the sinoatrial node fires at a rate of <60/min. SB is classified as symptomatic if, in addition to a heart rate <60/min, the client experiences such symptoms as dizziness, syncope, chest pain, and hypotension. The clinical significance of SB depends on how the client tolerates it. The client with symptomatic SB is first treated with atropine. If atropine is ineffective, transcutaneous pacing or an infusion of dopamine or epinephrine is considered. A permanent pacemaker may be needed. If SB is the result of a medication (eg, beta blocker, digoxin), the drug may need to be held, discontinued, or given in a reduced dosage. (Option 1) Complete heart block, or 3rd-degree atrioventicular (AV) block, is a form of AV dissociation in which no impulses from the atria are conducted to the ventricles. The atria are stimulated and contract independently of the ventricles. The ventricular rhythm is an escape rhythm. (Option 2) In 1st-degree AV block, every impulse is conducted to the ventricles, but the time of AV conduction is prolonged. This is evidenced by a prolonged PR interval of >0.20 second. (Option 4) Sinus rhythm has a rate of 60-100/min. Educational objective: The nurse should be able to recognize SB on the ECG and assess for clinical significance (eg, chest pain, syncope, hypotension) in the client. Initial expected treatment for symptomatic clients includes atropine and transcutaneous pacing. Copyright © UWorld. All rights reserved. Test Id: 52036792 Question Id: 32256 (729561) The nurse is caring for a debilitated client with a percutaneous endoscopic gastrostomy (PEG) tube that was inserted 3 days ago for the long-term administration of enteral feedings and medications. While the nurse is preparing to administer the feeding, the tube becomes dislodged. What is the most appropriate intervention? Unordered Options Ordered Response 1. Insert a Foley catheter into the existing tract and inflate the balloon 2. Insert a small-bore nasointestinal tube to administer feedings and medications 3. Notify the health care provider who inserted the PEG tube 4. Reinsert the PEG tube into the existing tract immediately Explanation A PEG is a minimally invasive procedure performed under conscious sedation. Using endoscopy, a gastrostomy tube is inserted through the esophagus into the stomach and then pulled through an incision made in the abdominal wall. To keep it secured, the PEG tube has an outer bumper and an inner balloon or bumper. The tube's tract begins to mature in 1-2 weeks and is not fully established until 4-6 weeks. It begins to close within hours of tube dislodgement. The nurse should notify the health care provider who placed the PEG tube as early dislodgement (ie, <7 days from placement) requires either surgical or endoscopic replacement (Option 3). (Options 1 and 4) The insertion of a Foley catheter or immediate reinsertion of the PEG tube should not be attempted because the tube's tract is only 3 days old (immature). A reinserted tube could be placed inadvertently into the peritoneal cavity, leading to serious consequences such as peritonitis and sepsis. Therefore, these are not the most appropriate interventions. (Option 2) Small-bore nasointestinal tubes are used for short-term rather than long- term administration of enteral feedings. They are prone to clogging from enteral feedings, undissolved medications, and inadequate tube flushes. They can also kink, coil, and become dislodged by coughing and may require frequent reinsertion. Therefore, they are not the most appropriate intervention. Educational objective: A PEG tube's tract begins to mature in 1-2 weeks and is fully established in 4-6 weeks. Tube dislodgement <7 days from placement requires surgical or endoscopic replacement. Attempting to reinsert a tube through an immature tract can result in improper placement into the peritoneal cavity, leading to peritonitis and sepsis. Test Id: 52036792 Question Id: 30186 (729561) A client diagnosed with septic shock has an upward-trending glucose level (180-225 mg/dL [10.0-12.5 mmol/L]) requiring control with insulin. The client's spouse asks why insulin is needed as the client is not a diabetic. What is the most appropriate response by the nurse? Unordered Options Ordered Response 1. "It is common for critically ill clients to develop type II diabetes. We give insulin to keep the glucose level under control (<140 mg/dL [7.8 mmol/L])." "The client was diabetic before, but you just didn't know it. We give insulin 2. to keep the glucose level in the normal range (70-110 mg/dL [3.9-6.1 mmol/L])." 3. "The increase in glucose is a normal response to stress by the body. We give insulin to keep the level at 140-180 mg/dL (7.8-10.0 mmol/L)." "This increase is common in critically ill clients and affects their ability to 4. fight off infection. We give insulin to keep the glucose level in the normal range (70-110 mg/dL [3.9-6.1 mmol/L])." Explanation Stress-induced hyperglycemia (gluconeogenesis) can occur in hospitalized clients in relation to surgery, trauma, acute illness, and infection. Hyperglycemia (glucose level >140 mg/dL [7.8 mmol/L]) affects both diabetic and non-diabetic hospitalized clients, especially those who are critically ill. Approximately 80% of clients in the intensive care unit who develop hyperglycemia have no history of diabetes before admission. Hyperglycemia is associated with increased risk of complications (eg, health care- associated infection, increased length of stay, acute kidney injury). To minimize complications and avoid hypoglycemia, the recommended glucose target range for critically ill clients is 140-180 mg/dL [7.8-10.0 mmol/L]. For non-critically ill clients, <140 mg/dL [7.8 mmol/L] fasting and <180 mg/dL [10.0 mmol/L] random blood glucose are recommended. (Option 1) Hospital hyperglycemia is not a direct cause of type II diabetes mellitus. In the non-diabetic client, the glucose level usually returns to normal after resolution of the disease process and/or discontinuation of steroid medications. A target glucose range of <140 mg/dL [7.8 mmol/L) is not recommended for this client. (Option 2) The prevalence of diabetes in hospitalized clients is high (about 1 in 4) and may be an undiagnosed pre-existing condition. A normal-range glucose level (70-110 mg/dL [3.9-6.1 mmol/L]) is not the recommended target range in this client due to the risk of hypoglycemia (with aggressive control) and worse outcomes. (Option 4) Although hyperglycemia does affect the ability to fight infection, 70-110 mg/dL [3.9-6.1 mmol/L] is not the recommended target range for this client. Educational objective: Stress-induced hyperglycemia causes complications in the hospitalized client. To minimize complications, the recommended target glucose range for critically ill clients is 140-180 mg/dL [7.8-10.0 mmol/L]. For non-critically ill clients, <140 mg/dL (7.8 mmol/L) fasting and <180 mg/dL (10.0 mmol/L) random blood glucose are recommended. Test Id: 52036792 Question Id: 30154 (729561) The nurse cares for an elderly client with type II diabetes who was diagnosed with diabetic retinopathy. Which statement by the client requires the most immediate action by the nurse? Unordered Options Ordered Response 1. "Half of my vision looks like it’s being blocked by a curtain." 2. "I have to use reading glasses to see small print." 3. "My vision seems cloudy and I notice a lot of glare." 4. "The colors don’t seem as bright as they used to." Explanation Chronic hyperglycemia can cause microvascular damage in the retina, leading to diabetic retinopathy, the most common cause of new blindness in adults. Option 1 indicates a retinal detachment requiring emergency management. A partial retinal detachment may be painless and cause symptoms such as a curtain blocking part of the visual field, floaters or lines, and sudden flashes of light. An unrepaired complete retinal detachment can cause blindness. (Option 2) The need for reading glasses is associated with presbyopia and is a common, nonemergency, age-related visual disorder. (Option 3) Cloudy vision with a glare is associated with a cataract, a nonemergency, age-related visual disorder. (Option 4) Although decreased vibrancy of colors is a sign of diabetic retinopathy and requires intervention, it is not indicative of a partial or complete retinal detachment; therefore, it is not an emergency. Educational objective: Chronic hyperglycemia can cause microvascular damage in the retina, leading to diabetic retinopathy, the most common cause of new blindness in adults. A partial retinal detachment may be painless and cause symptoms such as a curtain blocking part of the visual field, floaters or lines, and sudden flashes of light. Test Id: 52036792 Question Id: 30431 (729561) A client comes to the emergency department with diplopia and recent onset of nausea. Which statement by the client would indicate to the nurse that this is an emergency? Unordered Options Ordered Response 1. "I am very tired, and it's hard for me to keep my eyes open." 2. "I don't feel good, and I want to be seen." 3. "I have not taken my blood pressure medicine in over a week." 4. "I have the worst headache I've ever had in my life." Explanation A ruptured cerebral aneurysm is a surgical emergency with a high mortality rate. Cerebral aneurysms are usually asymptomatic unless they rupture; they are often called "silent killers" as they may go undetected for many years before rupturing without warning signs. The distinctive description of a cerebral aneurysm rupture is the abrupt onset of "the worst headache of my life" that is different from previous headaches (including migraines). Immediate evaluation for a possible ruptured aneurysm is critical for any client experiencing a severe headache with changes in or loss of consciousness, neurologic deficits, diplopia, seizures, vomiting, or a stiff neck. Early identification and prompt surgical intervention help increase the chance for survival. (Options 1, 2, and 3) A change in level of consciousness, increased blood pressure, or a feeling of illness should be investigated but alone does not indicate an emergency. Educational objective: Sudden onset of a severe headache described as "the worst headache of my life" is characteristic of a ruptured cerebral aneurysm and should be treated as an emergency. Test Id: 52036792 Question Id: 30220 (729561) The nurse in an outpatient clinic receives a blood test report of moderately elevated thyroid-stimulating hormone (TSH) and markedly decreased T3 and T4 levels. Which signs and/or symptoms should be expected in the client's evaluation? Select all that apply Unordered Options Ordered Response 1. Cold intolerance 2. Constipation 3. Forgetfulness 4. Hair loss 5. Warm, moist skin 6. Weight loss Explanation Hypothyroidism is a thyroid disorder characterized by thyroid hormone deficit (low T3 and T4). TSH is elevated due to compensatory increase from pituitary. Hypothyroidism affects almost every body system and is predominately associated with a slow metabolic rate. Some common manifestations include the following: 1. Decreased gut motility leading to constipation 2. Cool and pale skin due to decreased blood flow; hyperkeratosis results in dry and rough skin 3. Brittle nails and hair; hair loss due to poor blood supply 4. Bradycardia from low metabolic state 5. Joint pains and muscle aches are common 6. Clients can develop dementia and depression due to mental slowing 7. Cold intolerance characteristic 8. Modest weight gain (Options 5 and 6) Weight loss; heat intolerance; shakiness; diarrhea; and warm, moist skin are symptoms associated with hyperthyroidism or an increased metabolic rate. Educational objective: Signs and symptoms of hypothyroidism (a thyroid hormone deficit) are associated with a low metabolic rate. Weight gain, constipation, dry skin, hair loss, cold intolerance, bradycardia, mental slowing (dementia and depression), and anemia are some of the most common manifestations. Test Id: 52036792 Question Id: 30219 (729561) An elderly client with hypothyroidism is brought to the emergency department for depressed mental status. The client lives alone but has not taken medications for several months or seen a health care provider. Which action should the nurse take first? Click on the exhibit button for additional information. Unordered Options Ordered Response 1. Administer IV levothyroxine 2. Check serum thyroid-stimulating hormone, T3 and T4 3. Place a warming blanket on the client 4. Prepare for endotracheal intubation Explanation Myxedema coma is a complication associated with progression of symptoms of hypothyroidism from lethargy and mental sluggishness to a coma state. This client has hypothermia, bradycardia, hypotension, and depressed mental status. Hypothyroidism can also cause hypoventilation due to central depression of respiratory drive, respiratory muscle fatigue, and mechanical obstruction by a large tongue. This client exhibits signs of acute respiratory distress (increased respirations, very low oxygen saturation). Therefore, life-saving measures to facilitate respiratory support, such as mechanical ventilation, must be implemented first. Other treatments include thyroid hormone replacement with levothyroxine (Synthroid) IV push (Option 1), heating warming the client with a warming blanket (Option 3), and frequent diagnostics of the thyroid, including a serum thyroid panel (Option 2). Educational objective: Myxedema coma is a complication associated with progression of hypothyroidism symptoms. The highest-priority intervention is respiratory support for the client exhibiting signs of acute respiratory distress. Test Id: 52036792 Question Id: 30747 (729561) The nurse is caring for a client with cirrhosis. Assessment findings include ascites, peripheral edema, shortness of breath, fatigue, and generalized discomfort. Which interventions would be appropriate for the nurse to implement to promote the client's comfort? Select all that apply. Unordered Options Ordered Response 1. Encourage adequate sodium intake 2. Place client in semi-Fowler position 3. Place client in Trendelenburg position 4. Provide alternating air pressure mattress 5. Use music to provide a distraction You answered this question correctly. Explanation In a client with cirrhosis and ascites, discomfort is often due to pressure of the fluid on the surrounding organs. Shortness of breath occurs due to the upward pressure exerted by the abdominal ascites on the diaphragm, which restricts lung expansion. Positioning the client in semi-Fowler or Fowler position can promote comfort, as this position can reduce the pressure on the diaphragm (Option 2). In semi-Fowler position, the head of the bed is elevated 30-45 degrees; in Fowler position, elevation is 45-60 degrees. Side-lying with the head elevated can also be a position of comfort for the client with ascites as it allows the heavy, enlarged abdomen to rest on the bed, reducing pressure on internal organs and allowing for relaxation. Meticulous skin care is a priority due to the increased susceptibility of skin breakdown from edema, ascites, and pruritus. It is important to use a specialty mattress and implement a turning schedule of every 2 hours (Option 4). A distraction can take the client's mind off the current symptoms and may also help promote comfort in many different situations. Some of these distractions include listening to music, watching television, playing video games, or taking part in hobbies (Option 5). (Option 1) This client has ascites and peripheral edema; higher levels of fluid or sodium intake can worsen these conditions. (Option 3) In Trendelenburg position, the bed is tilted with the head lower than the legs. This position is contraindicated in the client with ascites, as it may exacerbate shortness of breath by causing the abdominal ascites to push upward on the diaphragm, restricting lung expansion. Educational objective: The client with discomfort and shortness of breath due to ascites should be positioned in the semi-Fowler or Fowler position to promote comfort and lung expansion. Music and other methods of distraction may also promote comfort. Meticulous skin interventions (eg, specialty mattress, turning schedule) are important to prevent tissue breakdown. Test Id: 52036792 Question Id: 30818 (729561) A previously healthy client is hospitalized with left lower lobe (LLL) bacterial pneumonia. The nurse assesses chest pain with inspiration, productive cough of thick rusty sputum, and LLL fine inspiratory crackles and low-pitched expiratory wheezing. Which of the medications that the health care provider prescribes should the nurse question? Unordered Options Ordered Response 1. Furosemide 20 mg intravenous (IV) push every day 2. Guaifenesin ER 600 mg PO every 12 hours 3. Ibuprofen 600 mg PO every 6 hours PRN 4. Levofloxacin 500 mg IV every day Explanation Medications commonly prescribed to treat bacterial pneumonia include antibiotics, expectorants, mucolytics, antipyretics, analgesics, and anti-inflammatories. Furosemide (Lasix) is a diuretic and is not appropriate for treating the fine crackles associated with pneumonia. The crackles result from alveolar filling and atelectasis, not from heart failure or pulmonary edema. (Option 2) Extended-release guaifenesin (Mucinex) is an expectorant medication that increases respiratory fluids and thins secretions to facilitate mobilization and expectoration. It is appropriate to prescribe in clients who have pneumonia with a productive cough and low-pitched wheezing (rhonchi). (Option 3) Ibuprofen is an anti-inflammatory medication administered to relieve pleuritic chest pain associated with pneumonia and is appropriate to prescribe in clients with pneumonia. (Option 4) Recommended antibiotic therapy for hospitalized clients with community- acquired streptococcal pneumonia includes monotherapy with a fluoroquinolone or combined therapy with a macrolide (eg, azithromycin [Zithromax]) plus a beta-lactam (eg, a cephalosporin [ceftriaxone]) for 1-2 days before transitioning to oral antibiotics. Educational objective: Medications prescribed to treat hospitalized clients with community-acquired bacterial pneumonia include IV antibiotics, expectorants, mucolytics, antipyretics, analgesics, and anti-inflammatories. Copyright © UWorld. All rights reserved. Test Id: 52036792 Question Id: 30856 (729561) Which health history information would be most important for the nurse to obtain when assessing a client with suspected bladder cancer who reports painless hematuria? Unordered Options Ordered Response 1. Family risk factors 2. Industrial chemical exposure 3. Tobacco use 4. Usual diet Explanation The tell-tale symptom of bladder cancer, seen in >75% of cases, is painless hematuria; the client will report seeing blood in the urine with no associated pain. As with many other types of cancer, the primary cause of bladder cancer is cigarette smoking or other tobacco use (Option 3). Poorer outcomes are seen with increased length of time as a smoker and higher number of packs per day. (Option 1) Clients who have family members with bladder cancer have an increased risk of developing bladder cancer themselves; however, the primary risk factor is tobacco use. (Option 2) Occupational carcinogen exposure is the second most common risk factor. Occupational exposures include printing, iron and aluminum processing, industrial painting, metal work, machining, and mining. Clients are exposed to carcinogens through direct skin contact and inhalation (aerosols and vapors). (Option 4) Consuming a high-fat diet and using artificial sweeteners are risk factors for developing bladder cancer, but they are not the primary cause. Educational objective: Painless hematuria is the most common presenting symptom of bladder cancer. Cigarette smoking or other tobacco use is the primary risk factor. Test Id: 52036792 Question Id: 33759 (729561) The nurse reviews prescriptions for assigned adult clients. Which prescription should the nurse question? Unordered Options Ordered Response 0.45% sodium chloride (NaCl) solution prescribed for a client with 1. syndrome of inappropriate antidiuretic hormone secretion who has a sodium level of 120 mEq/L (120 mmol/L) 2. 0.9% NaCl solution prescribed for a client with gastrointestinal bleeding who has a hemoglobin level of 8.9 g/dL (89 g/L) 3. 1,000 mL bolus of 0.9% NaCl solution prescribed for a client with septic shock who has a white blood cell count of 18,000/mm3 (18.0 × 109/L) 4. Lactated Ringer's solution prescribed for a male client with hypovolemic shock and a thermal burn who has a hematocrit level of 56% (0.56) Explanation Syndrome of inappropriate antidiuretic hormone secretion (SIADH) is associated with increased water reabsorption and excessive intra- and extracellular fluid, which result in hypervolemia from fluid retention and dilutional hyponatremia. In the setting of SIADH, the nurse should question a prescription for a hypotonic solution (eg, 0.45% NaCl; or dextrose water) as it would worsen the fluid and electrolyte imbalance. A prescription for fluid restriction and a hypertonic IV solution (eg, 3% NaCl) administered in small quantities would be appropriate to shift fluid back into the vascular compartment and correct hyponatremia. (Option 2) Isotonic fluids (eg, normal saline) are appropriate for clients with volume deficit such as those with gastrointestinal bleeding. (Option 3) Septic shock involves an inflammatory response to pathogens that leads to massive vasodilation and increased capillary permeability, resulting in intravascular hypovolemia and severe hypotension. An isotonic solution (eg, 0.9% NaCl) bolus is prescribed to expand intravascular volume and increase blood pressure. (Option 4) A burn injury causes tissue damage and increased capillary permeability; this leads to fluid and electrolyte losses related to evaporation and intravascular fluid shifts into the interstitial tissue, which result in hypovolemia, hemoconcentration (eg, hematocrit >53% [0.53]), and hypotension. An isotonic solution (eg, lactated Ringer's) is prescribed to replace fluid and electrolyte losses. Educational objective: Syndrome of inappropriate antidiuretic hormone secretion is associated with hypervolemia and dilutional hyponatremia. Fluid restriction and hypertonic IV solutions (eg, 3% saline) are prescribed to correct hyponatremia. Test Id: 52036792 Question Id: 31913 (729561) A client is admitted to the cardiac care unit with atrial fibrillation. Vital signs are shown in the exhibit. Which prescription should the nurse perform first? Click on the exhibit button for additional information. Unordered Options Ordered Response 1. Administer diltiazem 20 mg IVP 2. Administer rivaroxaban 20 mg PO 3. Draw blood for a thyroid function test 4. Send the client for echocardiogram Explanation Atrial fibrillation is characterized by a disorganization of electrical activity in the atria due to multiple ectopic foci. It results in loss of effective atrial contraction and places the client at risk for embolic stroke due to thrombi formed in the atria from stasis of blood. During atrial fibrillation, the atrial rate may be increased to 350-600/min. The ventricular response (pulse rate) can vary. The higher the ventricular rate, the more likely the client will have symptoms of decreased cardiac output (ie, hypotension). Ventricular rate control is a priority in clients with atrial fibrillation. This client has an irregular heart rate of 140/min and is not currently hypotensive. However, if the high ventricular response is allowed to continue, it is likely that the client will begin to show signs and symptoms of decreased cardiac output such as hypotension. Therefore, giving the client diltiazem (a calcium channel blocker) is the priority as its purpose is to decrease the ventricular response rate to <100/min. Other medications such as beta blockers (metoprolol) or digoxin may also be used to control the ventricular rate. (Option 2) Anticoagulants (eg, rivaroxaban [Xarelto], dabigatran [Pradaxa], apixaban [Eliquis], and warfarin) are used for long-term prevention of atrial thrombus and embolic complications. This is not a priority. (Option 3) The HCP will investigate possible causes of the atrial fibrillation; one of these is an overactive thyroid gland (hyperthyroidism). The thyroid function test would be useful for confirmation, but it is not a priority. (Option 4) An echocardiogram can be obtained once the rate is controlled, but it is not a priority. Educational objective: Ventricular rate control is a priority in the client with atrial fibrillation; therefore, the nurse should administer the medication (diltiazem, metoprolol, or digoxin) that will accomplish this first. Test Id: 52036792 Question Id: 30735 (729561) The nurse should plan to teach which client about the need for prophylactic antibiotics prior to dental procedures? Unordered Options Ordered Response 1. Client who had a large anterior wall myocardial infarction (MI) with subsequent heart failure 2. Client who had a mitral valvuloplasty repair 3. Client with a mechanical aortic valve replacement 4. Client with mitral valve prolapse with regurgitation Explanation Certain individuals should receive prophylactic antibiotics prior to dental procedures to prevent infective endocarditis (IE). These include the following: • Prosthetic heart valve or prosthetic material used to repair heart valve • Previous history of IE • Some forms of congenital heart disease o Unrepaired cyanotic congenital defect o Repaired congenital defect with prosthetic material or device for 6 months after procedure o Repaired congenital defect with residual defects at the site or adjacent to the site of a prosthetic patch or device • Cardiac transplantation recipients who develop heart valve disease (Option 1) The client with acute MI and heart failure is not at risk for IE. (Option 2) The client with mitral valve repair without the use of prosthetic material is at low risk for IE. (Option 4) The client with mitral valve prolapse with or without regurgitation, or aortic valve disease does not require prophylaxis for IE. Educational objective: Clients with any form of prosthetic material in their heart valves or who have unrepaired cyanotic congenital heart defect or prior history of IE should take prophylactic antibiotics prior to dental procedures to prevent development of IE. Test Id: 52036792 Question Id: 31413 (729561) A client is 1-day postoperative abdominoplasty and is discharged to go home with a Jackson-Pratt (JP) closed-wound system drain in place. The nurse teaches the client how to care for the drain and empty the collection bulb. Which statement indicates that the client needs further instruction? Unordered Options Ordered Response 1. "I'll empty the JP bulb when it is totally full so that I don't have to unplug it so many times." 2. "I'll pull the plug on the JP bulb and pour the drainage into the measurable specimen cup." 3. "I'll squeeze the JP bulb from side-to-side as I hold it in my hand." 4. "While the JP bulb is totally compressed, I'll clean the spout with alcohol and replace the plug." Explanation It is common for clients to be discharged with a JP closed-wound surgical drain in place after abdominal and breast reconstruction surgery. The purpose of the drain is to prevent fluid buildup in a closed space, which can put tension on the suture line and compromise the integrity of the incision, increase the risk for infection, and decrease wound healing. The general procedure for emptying the drainage device includes the following steps in order: • Perform hand hygiene as asepsis must be maintained to prevent the transmission of microorganisms even though there is less chance of bacteria entering the wound using a closed-wound drainage device (eg, JP, Hemovac) than an open-drain device (eg, Penrose) • Pull the plug on the bulb to open the device and pour the drainage into a small, calibrated container (eg, plastic water cup, urine specimen container) as this facilitates recording accurate drainage output (Option 2) • Empty the device every 4-12 hours unless it is 1/2 to 2/3 full before then because as the small capacity bulb (100 mL) fills, the amount of negative pressure in the bulb decreases (Option 1) • Compress the empty bulb by squeezing it from side-to-side with 1 or 2 hands until it is totally collapsed. Although the reservoir can be collapsed by pressing the bottom towards the top, compressing the sides of the reservoir (bulb) is recommended as it is more effective in establishing negative pressure (Option 3) • Clean the spout on the bulb with alcohol and replace the plug when it is totally collapsed to restore negative pressure (Option 4) Educational objective: A closed-wound drain device (eg, JP, Hemovac) is used to prevent fluid buildup at the surgical wound site and promote healing. Empty the device every 4-12 hours unless it is 1/2 to 2/3 full before then. Drainage tube patency and negative pressure in the reservoir (bulb) must be maintained to provide adequate drainage. Test Id: 52036792 Question Id: 30771 (729561) The nurse is caring for an adolescent client diagnosed with type 1 diabetes. The client exhibits hot, dry skin and a glucose level of 350 mg/dL (19.4 mmol/L). Arterial blood gases show a pH of 7.27. STAT serum chemistry labs have been drawn. Cardiac monitoring shows a sinus rhythm with peaked T waves, and the client has minimal urine output. What is the nurse's next priority action? Unordered Options Ordered Response 1. Administer IV regular insulin 2. Administer normal saline infusion 3. Obtain urine for urinalysis 4. Request prescription for potassium infusion Explanation This client has diabetic ketoacidosis (DKA). All clients with DKA experience dehydration due to osmotic diuresis. Prompt and adequate fluid therapy restores tissue perfusion and suppresses the elevated levels of stress hormones. The initial hydrating solution is 0.9% saline infusion. (Option 1) Insulin therapy should be started after the initial rehydration bolus as serum glucose levels fall rapidly after volume expansion. (Option 3) Urinalysis is important but not a priority. (Option 4) Potassium should never be given until the serum potassium level is known to be normal or low and urinary voiding is observed. Peaked T waves indicate hyperkalemia in this client. Clients with insulin deficiency frequently have increased serum potassium levels due to the extracellular shift despite having total body potassium deficit from urinary losses. Once insulin is given, serum potassium levels drop rapidly, often requiring potassium replacement. Potassium is never given as a rapid IV bolus, as cardiac arrest may result. Educational objective: Clients with diabetic ketoacidosis and hyperosmolar hyperglycemic state require IV normal saline as a priority due to severe dehydration. Once fluids are given as a bolus, insulin is initiated. The serum potassium levels can be elevated in the initial stages despite a low total body potassium. Potassium repletion is started once the serum potassium levels are normalized or trending low (from elevated levels). Test Id: 52036792 Question Id: 32379 (729561) A client tells the nurse of wanting to lose 20 lb (9 kg) in time for the client's daughter's wedding, which is 16 weeks away. How many calories (kcal) will the client have to eliminate from the diet each day to meet this goal? Unordered Options Ordered Response 1. 450 kcal/day 2. 625 kcal/day 3. 860 kcal/day 4. 1,000 kcal/day Explanation A reduction or energy expenditure of 3,500 calories (kcal) will result in a weight loss of 1 lb (0.45 kg). To lose 20 lb (9 kg), the client needs to reduce intake by a total of 70,000 kcal (3500 kcal x 20 lb [9 kg] = 70,000 kcal). Over a period of 16 weeks, this would require a daily reduction of: 625 kcal (70,000 kcal / [16 weeks x 7 days] = 625 kcal/day) Adding an exercise regimen to the client's daily routine would facilitate additional weight loss and/or reduce the need for severe caloric restriction. (Option 1) Reducing intake by 450 kcal/day over 16 weeks would result in a weight loss of 14.5 lb (6.5 kg). (Option 3) Reducing intake by 860 kcal/day over 16 weeks would result in a weight loss of 27.5 lb (12.4 kg). (Option 4) Reducing intake by 1000 kcal/day over 16 weeks would result in a weight loss of 32 lb (14.5 kg). Educational objective: A reduction or energy expenditure of 3500 calories (kcal) will result in a weight loss of Test Id: 52036792 Question Id: 30285 (729561) The nurse is assisting with a colorectal cancer screening using the guaiac fecal occult blood test (gFOBT). Place the steps for completing this test in the correct sequence. All options must be used. Unordered Options Your Response/Correct Res Document the results in the electronic medical record Obtain supplies, wash hands, and apply Obtain supplies, wash hands, and apply non-sterile Open the slide's flap and use the wooden gloves 2 separate stool samples to the boxes on Open the back of the slide and apply 2 drops of Wait 3-5 minutes developing solution to the boxes on the slide Open the back of the slide and apply 2 dr Open the slide's flap and use the wooden applicator to apply 2 separate stool samples to the boxes on the slide solution to the boxes on the slide Wait 30-60 seconds Wait 30-60 seconds Document the results in the electronic me Wait 3-5 minutes Explanation The gFOBT is used to assess for microscopic blood in the stool and as a screening tool for colorectal cancer. The steps for collecting a gFOBT sample are as follows: 1. The nurse first communicates the purpose of the test to the client. An interview is conducted to assess recent ingestion (within last 3 days) of red meat or vitamin C supplements as these may interfere with the test and produce false results. A medication history is taken related to use of medications that might interfere with test results, including aspirin, anticoagulants, iron, ibuprofen, and corticosteroids. 2. Obtain supplies, wash hands, and apply non-sterile gloves (Option 2). Supplies include FOB slide, wooden applicator (tongue blade), hemoccult developer, and clean gloves. 3. Open the slide's flap and use the wooden applicator to apply 2 separate stool samples to the boxes on the slide. Collect the fecal samples from 2 different areas of the specimen as some portions of the stool may not contain microscopic blood (Option 4). 4. Close slide cover and allow the stool specimen to dry for 3-5 minutes (Option 6). 5. Open the back of the slide and apply 2 drops of developing solution to the boxes on the slide (Option 3). 6. Assess the color of the hemoccult slide paper after 30-60 seconds. If the test paper turns blue, (a positive guaiac result), the stool contains microscopic blood (Option 5). 7. Place used gloves and wooden applicator in appropriate waste receptacle. 8. Perform hand hygiene. 9. Document the test results in the electronic medical record (Option 1). Educational objective: gFOBT detects microscopic blood in the stool and is useful as a colorectal cancer screening test. A blue color on the hemoccult slide paper is a positive result and indicates the presence of blood in the stool. Test Id: 52003011 Question Id: 30261 (729561) A client with mitral valve disease is experiencing uncontrolled atrial fibrillation (AF) for 3 days that has been unresponsive to drug therapy. The client is scheduled for electrical cardioversion. What other procedure or test does the nurse anticipate that this client will require? Unordered Options Ordered Response 1. Chest x-ray 2. Exercise stress test 3. Insertion of a central venous access 4. Transesophageal echocardiogram (TEE) Explanation Electrical cardioversion is a treatment modality considered for AF that has been unresponsive to drug therapy. AF (rapid, irregular atrial contractions) results in ineffective atrial kick and predisposes to thrombus formation (blood clots) in the left atrium. If a client is in AF for more than 48 hours, anticoagulation therapy is needed for 3-4 weeks before cardioversion. Anticoagulation therapy is necessary as cardioversion may dislodge an atrial thrombus, putting the client at risk for a stroke or other sequelae of thromboembolism. If 4 weeks of anticoagulation is not an option, TEE must be performed prior to cardioversion. (Option 1) A chest x-ray does not help identify clots in the atria. (Option 2) An exercise stress test is not indicated in a client who is experiencing dysrhythmia. It also does not help identify clots. (Option 3) Central venous access is not necessary for this procedure, although at least 2 large-bore intravenous sites would be helpful. Educational objective: A TEE is indicated prior to cardioversion for a client who has been in AF for more than 48 hours, as cardioversion is contraindicated in the presence of an atrial thrombus. Test Id: 52003011 Question Id: 30749 (729561) A client diagnosed with end-stage renal disease comes to the dialysis clinic for treatment. Which actions should the nurse take to prepare the client for hemodialysis? Select all that apply. Unordered Options Ordered Response 1. Administer subcutaneous heparin to decrease clotting during dialysis 2. Administer the client's morning doses of carvedilol and lisinopril 3. Check the client's medical records to determine the last post- dialysis weight 4. Obtain a set of client vital signs and the client's current weight 5. Palpate the fistula in the client's arm for a thrill and auscultate for a bruit Explanation Prior to dialysis treatment, the nurse should assess the client's fluid status (weight, blood pressure, peripheral edema, lung and heart sounds), vascular access (arteriovenous fistula, arteriovenous grafts), and vital signs (Option 4). The amount of fluid removed (ultrafiltration) is determined by calculating the difference between the last post-dialysis weight and the client's current pre-dialysis weight (Option 3). After the client is connected to the dialysis machine, IV heparin is added to the blood from the client to prevent clotting that can occur when blood contacts a foreign substance. Giving subcutaneous heparin prior to initiation is not necessary (Option 1). (Option 2) During dialysis, excess fluid is removed, making the client prone to hypotension. In addition, medications are removed from the blood during hemodialysis, making them ineffective. Many medications that are taken once daily can be held until after the dialysis treatment to prevent their removal. If blood pressure medications are given prior to dialysis, the client can develop hypotension during the dialysis and then uncontrolled hypertension (decreased drug concentrations). (Option 5) Arteriovenous fistulas are created by anastomosing an artery to a vein; a thrill can be felt when palpating the fistula, and a bruit can be heard during auscultation when the fistula is functioning properly. Educational objective: The nurse is responsible for assessing the client diagnosed with end-stage renal disease for risks associated with dialysis. These risks include medication removal, hemodialysis access dysfunction, hypotension, and fluid and Test Id: 52003011 Question Id: 30232 (729561) The nurse in the intensive care unit cares for a client with primary adrenocortical insufficiency (Addison's disease). The client reports nausea and abdominal pain. The blood pressure suddenly drops from 120/74 mm Hg to 88/48 mm Hg, heart rate increases from 80 to 100/min, and the client appears confused. Which action should the nurse take first? Unordered Options Ordered Response 1. Administer as-needed dose of hydrocortisone intravenous (IV) push 2. Complete a head-to-toe assessment to identify any sources of infection 3. Document the findings in the client’s electronic medical record 4. Take blood pressure sitting and standing to assess for orthostatic hypotension Explanation Addison's disease is adrenocortical insufficiency or hypofunction of the adrenal cortex. Addisonian crisis or acute adrenocortical insufficiency is a potentially life-threatening complication. Clients report nausea, vomiting, and abdominal pain. Signs of addisonian crisis include hypotension, tachycardia, dehydration, hyperkalemia, hyponatremia, hypoglycemia, fever, weakness, and confusion. Priority emergency management of addisonian crisis includes shock management, with fluid resuscitation using 0.9% normal saline and 5% dextrose; and administration of high-dose hydrocortisone replacement IV push. (Options 2, 3, and 4) Assessment and documentation are important components of the nursing process, but emergency treatment of an addisonian crisis is the priority action. Educational objective: Addisonian crisis is a potentially life-threatening complication of Addison's disease and commonly presents with abdominal pain, hypotension, and hypoglycemia. Emergency management includes shock management with fluid resuscitation using 0.9% normal saline and 5% dextrose, and administration of high-dose hydrocortisone replacement IV push Test Id: 52003011 Question Id: 30481 (729561) A client admitted to the cardiac care unit with markedly elevated blood pressure and heart failure is receiving a continuous infusion of nitroprusside. Thirty minutes later, the client appears pale with cold, clammy skin and reports being lightheaded. Which is the priority nursing action? Unordered Options Ordered Response 1. Auscultate the client's lungs 2. Check the client's capillary refill 3. Measure the client's blood pressure 4. Review the client's electrocardiogram (ECG) Explanation Sodium nitroprusside is a highly potent vasodilator (both venous and arteriolar). Venous dilation reduces preload (volume of blood in ventricles at the end of diastole), and arterial dilation reduces afterload (resistance ventricle must overcome to eject blood during systole). Sodium nitroprusside is commonly used in hypertensive emergencies and for conditions in which blood pressure control is of utmost importance (eg, aortic dissection, acute hypertensive heart failure). Nitroprusside begins to act within 1 minute and can produce a sudden and drastic drop in blood pressure (symptomatic hypotension) if not monitored properly. Therefore, the client's blood pressure should be monitored closely (every 5–10 minutes). This client's lightheadedness and cold clammy skin are likely due to hypotension. Nitroprusside metabolizes to cyanide, and clients with renal disease can occasionally develop fatal cyanide toxicity. (Option 1) Auscultation of lung sounds is a common assessment for the client in heart failure. However, in this client the signs and symptoms indicate hypotension and make checking the blood pressure a higher priority. (Option 2) Checking capillary refill can give the nurse information about perfusion status. Capillary refill may be prolonged and should be checked in this client, but after blood pressure is measured. (Option 4) The ECG of this client should be reviewed. The client is at risk for rhythm abnormalities, but because hypotension is the main adverse effect of nitroprusside, the blood pressure should take precedence. Educational objective: Sodium nitroprusside is given as an infusion for the short-term treatment of acute decompensated heart failure, especially in clients with markedly elevated blood pressure. It is a potent vasodilator and reduces preload and afterload. The main adverse effect is symptomatic hypotension, necessitating close monitoring of blood pressure. Test Id: 52003011 Question Id: 30308 (729561) An elderly client reports shortness of breath with activity for the past 2 weeks. The nurse reviews the admission laboratory results and identifies which value as the most likely cause of the client's symptoms? Unordered Options Ordered Response 1. Brain natriuretic peptide 70 pg/mL (70 pmol/L) 2. Hematocrit 21% (0.21) 3. Leukocytes 3,500/mm3 (3.5 x 109/L) 4. Platelets 105,000/mm3 (105 x 109/L) Explanation Hematocrit (Hct) is the percentage of red blood cells (RBCs) in a volume of whole blood. Hct and hemoglobin (Hgb) values are related (approximately 3 x Hgb = Hct); when one value is decreased, the other is also. This client likely has hemoglobin of 7 g/dL (70 g/L) (normal, 13.2-17.3 g/dL [132-173 g/L] for males and 11.7-15.5 g/dL [117- 155 g/L] for females). Hgb is a component of the RBC that carries oxygen to the body's tissues. A decrease in Hgb decreases oxygen-carrying capacity and transport to tissues. RBCs may be 100% saturated with oxygen at rest, but desaturation may occur with increased activity and oxygen demand in the presence of decreased Hct and Hgb. Manifestations associated with decreased oxygen transport include shortness of breath with activity, tachypnea, and tachycardia. (Option 1) Brain natriuretic peptide (BNP) >100 pg/mL (100 pmol/L) is considered elevated and indicates ventricular stretch (heart failure) as the cause of the dyspnea. This client has normal BNP levels, making heart failure an unlikely cause. (Option 3) The leukocyte count is decreased (normal, 4,000-11,000/mm3 [4.0-11.0 x 109/L]). Leukocytes play a role in protecting the body from disease. (Option 4) The platelet count is decreased (normal, 150,000-400,000/mm3 [150-400 x 109/L]). Platelets play a role in blood clotting. Educational objective: Hemoglobin is a component of red blood cells that carries oxygen to the body's tissues. In the presence of decreased hematocrit and hemoglobin, decreased oxygen-carrying capacity and transport occur. Manifestations associated with decreased oxygen transport include shortness of breath with activity, tachypnea, and tachycardia. Test Id: 52003011 Question Id: 31950 (729561) The nurse is caring for a client with liver cirrhosis who was admitted for cellulitis of the leg. Which assessments would the nurse perform to determine if the client's condition has progressed to hepatic encephalopathy? Select all that apply. Unordered Options Ordered Response 1. Ask if the client knows what day it is 2. Ask the client to extend the arms 3. Assess for telangiectasia (spider nevi) 4. Determine if the conjunctiva is jaundiced 5. Note amylase and lipase serum levels Explanation Hepatic encephalopathy (HE) is a frequent complication of liver cirrhosis. Precipitating factors include hypokalemia, constipation, gastrointestinal hemorrhage, and infection. It results from accumulation of ammonia and other toxic substances in blood. Clinical manifestations of HE range from sleep disturbances (early) to lethargy and coma. Mental status is altered, and clients are not oriented to time, place, or person (Option 1). A characteristic clinical finding of HE is presence of asterixis (flapping tremors of the hands). It is assessed by having the client extend the arms and dorsiflex the wrists (Option 2). Another sign is fetor hepaticus (musty, sweet odor of the breath) from accumulated digestive byproducts. (Option 3) Spider angiomas (eg, small, dilated blood vessels with bright red centers), gynecomastia, testicular atrophy, and palmar erythema are expected findings in cirrhosis due to altered metabolism of hormone in the liver. (Option 4) Jaundice occurs when bilirubin is 2-3 times the normal value. Jaundice can occur in hepatitis and tends to worsen in cirrhosis due to increasing functional derangement. It is not related specifically to encephalopathy. (Option 5) Amylase and lipase are enzymes from pancreatic tissue. Alanine aminotransferase and aspartate aminotransferase are liver enzymes. They would be elevated with hepatitis and are not unique to cirrhosis or HE. Elevated ammonia levels would be more specific to cirrhosis. Educational objective: HE manifests with sleep disturbances, altered mental status, and lethargy. Asterixis and elevated ammonia are characteristic of HE. Test Id: 52003011 Question Id: 31762 (729561) A home health nurse is assessing for complications in a client who has been using crutches for 2 weeks. Assessing for which finding is most important? Unordered Options Ordered Response 1. Biceps muscle spasm 2. Forearm swelling 3. Hand and wrist weakness 4. Shoulder range of motion Explanation Excessive and prolonged pressure on the axillae can cause localized damage to the radial nerve at the axilla. This leads to a reversible condition known as crutch paralysis, or palsy, which manifests as muscle weakness and/or sensory symptoms (tingling, numbness) of the arm, wrist, and hand. It is caused by crutches that are too long or by leaning on the top of the crutches when ambulating. Therefore, clients are taught to support body weight on the hands and arms, not the axillae, when ambulating to ensure that there is a 1-2 in (2.5-5 cm) space between the axilla and the axilla crutch pad. Crutches should be checked for proper length. (Option 1) Triceps muscle spasm can occur due to increased muscle use, especially in clients with decreased upper body strength. Triceps and biceps muscle spasms are not complications associated with crutch walking. (Option 2) Forearm swelling is not a common complication associated with crutch walking. In rare cases, arterial obstruction can cause ischemic symptoms. (Option 4) Restricted shoulder range of motion is not a major complication of crutch use. Educational objective: Clients are taught to support their body weight on the hands and arms, not the axillae, when ambulating with crutches. Prolonged and continual excessive pressure on the axillae can damage the radial nerve, resulting in crutch paralysis - muscle weakness and/or sensory symptoms over the forearm, wrist, and hand. Test Id: 52003011 Question Id: 30305 (729561) The nurse is caring for a client who has been admitted to the hospital for an acute exacerbation of heart failure. Blood pressure is 104/62 mm Hg, pulse is 96/min, respirations are 22/min, and oxygen saturation is 91%. Which of these findings supports the diagnosis of acute heart failure exacerbation? Unordered Options Ordered Response 1. B-type natriuretic peptide (BNP) 1382 pg/mL [1382 pmol/L] 2. Flat jugular veins when seated at a 45-degree angle 3. Sodium 150 mEq/L [150 mmol/L] 4. Urine output greater than 100 mL/hr Explanation Brain (or b-type) natriuretic peptide (BNP) is secreted in response to ventricular stretch and wall tension when cardiac filling pressures are elevated. The BNP level is used to differentiate dyspnea of heart failure from dyspnea of noncardiac etiology. The level of circulating BNP correlates with both severity of left ventricular filling pressure elevation and mortality. A normal BNP level is <100 pg/mL [<100 pmol/L]. The nurse would expect a high BNP in a client exhibiting symptoms of acute decompensated heart failure. (Option 2) Jugular veins should normally flatten and disappear as the client is raised to an upright position. Jugular venous distension present above a 45-degree seated position indicates fluid volume excess and elevated cardiac filling pressures that occur with heart failure. (Option 3) Normal sodium level is 135-145 mEq/L [135-145 mmol/L]. Serum sodium can be normal or low in heart failure clients. Low levels are due to dilution from excess free water. (Option 4) Urine output of 100 mL/hr should be adequate to maintain fluid volume status. Inadequate urine output may cause fluid retention and volume overload, precipitating an exacerbation of heart failure. A state of low cardiac output may also decrease renal perfusion, resulting in renal dysfunction and decreased urine output. Diuretic therapy is the mainstay treatment for fluid volume overload. The nurse should expect to see an increase in urine output in response to diuretic administration. Educational objective: The nurse should assess the BNP level in clients admitted with heart failure exacerbations. Elevated BNP levels indicate increased ventricular stretch and correlate with severity of heart failure and fluid volume overload. Heart failure clients may also present with jugular venous distension, low serum sodium, and decreased urine output. Test Id: 52003011 Question Id: 30703 (729561) A client undergoes transurethral resection of the prostate (TURP) for benign prostatic hyperplasia. The client has a 28 Fr, 30-mL balloon, 3-way Foley catheter with continuous bladder irrigation. Which assessment by the nurse is the best indication that the bladder irrigation is running at an adequate rate? Unordered Options Ordered Response 1. Blood pressure 120/80 mm Hg, pulse 80/min 2. Client has no bladder spasms 3. Intake 3200 mL, output 3000 mL 4. Output urine is light pink in color Explanation TURP involves the insertion of a rectoscope to excise obstructing prostate tissue. Continuous bladder irrigation is initiated after the procedure. The large catheter and balloon apply direct pressure on the bleeding tissue and allow urine to drain. A specific rate is not prescribed; the nurse is expected to adjust irrigation flow so that the urine remains light pink without clots. Typically, the irrigation rate will gradually decrease during the first 24 hours. (Option 1) Vital signs within normal limits indicate hemodynamic stability but not patency of the draining catheter from the irrigation. (Option 2) Painful bladder spasms are expected after TURP and catheter placement. The spasms are typically treated with belladonna-opium suppositories or other antispasmodics (eg, anticholinergics such as oxybutynin [Ditropan]). The nurse should remind the client to refrain from trying to void around the catheter as this can trigger the spasms. (Option 3) Overall, the intake and output should be equal (considering approximately 400-500 mL/day of insensible loss). If the negative balance is ≥500 mL, further assessment/intervention is needed. Fluid balance is not the best indicator of irrigation infusion rate in these clients. Educational objective: The rate of continuous bladder irrigation after TURP is adjusted to keep the urinary output light pink in color. Bladder spasms (treated with belladonna-opium suppositories) are expected after the procedure. Test Id: 52003011 Question Id: 31944 (729561) The emergency department nurse is assessing a client brought in after a car accident in which the client's head hit the steering column. Which assessment findings would indicate that the triage nurse should apply spinal immobilization? Select all that apply. Unordered Options Ordered Response 1. Breath smells of alcohol 2. Client disoriented to place 3. Client reports eyes burning 4. History of multiple sclerosis 5. Point tenderness over spine Explanation Spinal immobilization is not a benign procedure. An acronym to help determine the need for spinal immobilization is NSAIDs: N - Neurological examination. Focal deficits include numbness and decreased strength. S - Significant traumatic mechanism of injury A - Alertness. The client may be disoriented or have an altered level of consciousness (Option 2). I - Intoxication. The client could have impaired decision-making ability or lack awareness of pain (Option 1). D - Distracting injury. Another significant injury could distract the client from spinal pain. S - Spinal examination. Point tenderness over the spine or neck pain on movement (if there is no midline tenderness) may be present (Option 5). (Option 3) The sensation of burning eyes could be related to many issues and does not necessarily have a direct correlation to spinal trauma. (Option 4) There is no direct correlation of multiple sclerosis (autoimmune progressive nerve demyelinization) with the need for spinal immobilization. Educational objective: Indications for spinal immobilization include abnormal neurological findings, significant mechanism of injury, change in orientation or level of consciousness, intoxication, distracting injury, and point tenderness over the spine. Test Id: 52003011 Question Id: 32957 (729561) The office nurse receives 4 telephone messages. Which client should the nurse call back first? Unordered Options Ordered Response 1. 28-year-old female client who fell on ice yesterday and has low back pain and spasm 2. 42-year-old male client who developed sharp, burning leg pain radiating from buttock to knee after lifting heavy weights 3. 65-year-old female client 10 days post spinal fusion who has increased persistent back pain and fever of 101.2 F (38.4 C) 4. 70-year-old male client with peripheral vascular disease who has acute-onset abdominal pain radiating to the low back Explanation An abdominal aortic aneurysm (AAA) is a blood-filled bulge in the abdominal aorta caused by weakening in the vessel wall due to increased pressure. Risk factors include male sex, age >65, coronary artery and peripheral vascular diseases, hypertension, and family and smoking history. AAA dissection (blood leakage into a vessel tear) or rupture may manifest as acute-onset abdominal pain radiating to the back and is typically associated with symptoms of hemorrhagic shock (eg, decreased systolic pressure; increased, weak pulses; pallor). This client’s symptoms could indicate impending rupture, which can lead to life-threatening vascular hemorrhage. (Option 1) This client's pain is most likely musculoskeletal in origin and due to a lumbosacral strain. Although assessment and treatment (eg, nonsteroidal anti- inflammatory drugs, muscle relaxants) are necessary, this is not a life-threatening condition. (Option 2) This client's pain is most likely radicular (irritation of the sciatic nerve) in origin. Although neurovascular evaluation for a herniated disk (L5-S1) is necessary, this is not a life-threatening condition. (Option 3) This client's pain and fever can be associated with a postoperative infection in the bone and surrounding tissue (osteomyelitis). Although diagnosis and treatment with prescribed antibiotics are crucial to prevent sepsis, a potential massive hemorrhage is a higher priority. Educational objective: Clients with atherosclerotic vascular disease in one system (eg, stroke, peripheral vascular disease) are more likely to have undiagnosed, underlying atherosclerotic vascular disease in other areas (eg, coronary disease, aortic aneurysm). Evaluation and treatment of a suspected abdominal aortic aneurysm dissection or rupture are critical as a vascular bleed is potentially life-threatening. Test Id: 52003011 Question Id: 32083 (729561) The nurse is planning care for a client experiencing an acute attack of Meniere disease. Which action is a high priority to include in the plan of care? Unordered Options Ordered Response 1. Initiate fall precautions 2. Keep the emesis basin at bedside 3. Provide a quiet environment 4. Start intravenous fluids Explanation Meniere disease (endolymphatic hydrops) results from excess fluid accumulation inside the inner ear. Clients have episodic attacks of vertigo, tinnitus, hearing loss, and aural fullness. The vertigo can be severe and associated with nausea and vomiting. Clients report feelings of being pulled to the ground (drop attacks). During an attack, the client is treated with vestibular suppressants, including sedatives (eg, benzodiazepines such as diazepam), antihistamines (eg, diphenhydramine, meclizine), anticholinergics (eg, scopolamine), and antiemetics. The nurse's priority is to plan for client safety with fall precautions given the severe vertigo and use of sedating medications. Fall precautions include adjusting the bed to a low position with side rails up and instructing the client to call for help before getting up. Vertigo may be minimized by staying in a quiet, dark room and avoiding sudden head movements. The client should reduce stimulation by not watching television or looking at flickering lights. The client's diet should be salt restricted to prevent fluid buildup in the ear. (Option 2) An emesis basin should be provided at the bedside, but fall precautions are the priority. (Option 3) A quiet environment can help minimize vertigo. However, it is a lower priority than the fall precautions. (Option 4) Most clients with Meniere disease require parenteral fluids given the nausea and vomiting. However, these are not the highest priority. Educational objective: Clients with Meniere disease (endolymphatic hydrops) can have severe vertigo, tinnitus, hearing loss, and aural fullness. It is a priority for the nurse to institute safety measures, such as fall precautions, for these clients. They Test Id: 52003011 Question Id: 33405 (729561) The nurse is caring for a client with suspected Graves disease. Which assessment finding requires priority intervention? Unordered Options Ordered Response 1. Agitation and confusion 2. Heat intolerance 3. Pulse of 110/min, irregular rhythm 4. Red and bulging eyes Explanation Thyroid storm is a serious and potentially life-threatening emergency for clients with Graves disease. This condition occurs when the thyroid gland releases large amounts of thyroid hormone in response to stress (eg, trauma, surgery, infection). Characteristic features include tachycardia, hypertension, cardiac arrhythmias (eg, atrial fibrillation), and fever up to 104-106 F (40-41 C). Other findings include severe nausea, vomiting, anxiety, altered mentation, and seizures. (Option 2) Heat intolerance is an expected symptom in hyperthyroidism, including Graves disease. (Option 3) Tachycardia and arrhythmias (eg, atrial fibrillation) are commonly seen with hyperthyroidism of any cause, including Graves disease. These alone cannot differentiate whether the client has simple hyperthyroidism or life-threatening thyroid storm. (Option 4) Exophthalmos (protruding eyeball) is commonly seen in Graves disease. The eyelids do not close over the eyeballs properly, leading to excessive dryness and resultant corneal damage (exposure keratitis). Although it is important to treat exophthalmos, it is not immediately life-threatening. Educational objective: Thyroid storm is a life-threatening complication of Graves disease. Fever, altered mentation, and excess autonomic activity (eg, severe hypertension, tachycardia) are common. Early recognition and treatment are crucial. Copyright © UWorld. All rights reserved. Test Id: 52003011 Question Id: 31146 (729561) The nurse admits a client with newly diagnosed unstable angina. Which information obtained during the admission health history is most important for the nurse to report to the health care provider (HCP) immediately? Unordered Options Ordered Response 1. Drinks 6 cans of beers on the weekend 2. Gets up 4 times during the night to void 3. Smokes 1 pack of cigarettes daily 4. Uses sildenafil occasionally Explanation Sildenafil (Viagra) is a phosphodiesterase inhibitor used to treat erectile dysfunction. The use of sildenafil is most important for the nurse to report to the HCP. This must be communicated immediately as concurrent use of nitrate drugs (commonly prescribed to treat unstable angina) is contraindicated as it can cause life-threatening hypotension. Before any nitrate drugs can be administered, further action is necessary to determine when sildenafil was taken last (ie, half-life is about 4 hours). (Option 1) Clients do not always report the amount of alcohol they consume accurately. The nurse should monitor all clients for alcohol withdrawal syndrome as it is quite common in hospitalized clients. (Option 2) Getting up 4 times during the night to void can be associated with medication, an enlarged prostate gland, or drinking fluids at bedtime. Further action may be needed to determine the cause of the nocturia, but this is not the most significant information to report to the HCP. (Option 3) Smoking 1 pack of cigarettes daily needs to be addressed as tobacco causes vasoconstriction and decreased oxygen supply to the body tissues. Further action is needed regarding smoking cessation education. However, the client's tobacco history is not the most important information to report to the HCP. Educational objective: Nitrate drugs are prescribed to treat angina. The concurrent use sildenafil (Viagra) and nitrates is contraindicated as it can cause life-threatening hypotension. Test Id: 52003011 Question Id: 31041 (729561) The nurse is providing discharge teaching for a client who suffered full-thickness burns. Which statement by the client demonstrates a need for further instruction on the rehabilitation phase of a burn injury? Unordered Options Ordered Response 1. "I should avoid using lotion to prevent infection." 2. "I should perform range-of-motion exercises daily." 3. "I will avoid direct sun exposure for at least 3 months." 4. "I will wear pressure garments to minimize scars." Explanation The rehabilitation phase begins after the client's wounds have fully healed and lasts about 12 months. The initiation of this phase depends on the extent of the burns and the client's ability to care for themselves. Interventions in the rehabilitation phase are aimed at improving mobility and independence and minimizing the potential for long-term complications. These interventions include: • Counseling or other psychosocial support • Gentle massage with water-based lotion to alleviate itching and minimize scarring • Planning for reconstructive surgery • Pressure garments to prevent hypertrophic scars and promote circulation (Option 4) • Range-of-motion exercises to prevent contractures (Option 2) • Sunscreen and protective clothing to prevent sunburns and hyperpigmentation (Option 3) (Option 1) Daily application of water-based lotion is necessary to minimize scar formation and alleviate itching. Infection is not likely as the rehabilitation phase begins after the wounds are fully healed. Educational objective: The rehabilitation phase begins after the client's wounds are healed. The goals of this phase are to increase the client's ability to perform activities of daily living and prevent long-term complications. Test Id: 52003011 Question Id: 30787 (729561) The nurse administers IV vancomycin to a client with a methicillin-resistant staphylococcus aureus infection. Which actions are most appropriate for the nurse to perform? Select all that apply. Unordered Options Ordered Response 1. Ask the client to report symptoms of hematuria 2. Assess skin for flushing and red rash on face and torso 3. Draw trough serum concentration level after the infusion 4. Infuse medication over at least 60 minutes 5. Monitor blood pressure during the infusion 6. Observe for pain, redness, or swelling at the IV site every 30 minutes Explanation When administering IV vancomycin, the nurse should perform the following nursing actions to prevent possible complications: • Assess skin for flushing and red rash on face and torso. Red man syndrome is a nonallergic histamine reaction that is characterized by the sudden onset of severe hypotension, flushing, and/or maculopapular rash of the face, neck, chest, and upper extremities. • Draw prescribed trough serum concentration level. It is obtained immediately before infusing the fourth dose to help determine the appropriate maintenance dose. • Infuse medication over at least 60 minutes (≤10 mg/min). This slower rate minimizes development of thrombophlebitis, extravasation, and red man syndrome (>10%). • Monitor blood pressure during the infusion. Hypotension (>10%) or red man syndrome can occur if the medication is administered too quickly. • Observe for pain, redness, or swelling at the IV site every 30 minutes. IV vancomycin has an acid pH (<5) and is extremely irritating to the vein. It can cause thrombophlebitis (1-10%) and tissue necrosis if it extravasates. • Monitor for a hypersensitivity reaction and signs of anaphylaxis (rash, pruritus, laryngeal edema, and wheezing). • Ideally administer through a peripherally inserted central catheter or central venous catheter as the medication is a vesicant. It is often initiated through a peripheral IV site for short-term use. (Option 1) Cyclophosphamide (not vancomycin) infusion can cause hemorrhagic cystitis, resulting in hematuria. (Option 3) Serum trough vancomycin concentration is drawn immediately before (not after) the fourth dose. It is monitored to achieve and maintain an appropriate therapeutic drug level (ie, 10-20 mg/L for hemodynamically stable clients), to limit adverse effects (eg, nephrotoxicity/ototoxicity, with occurrence <1%), and to help determine effectiveness of the drug. Educational objective: Assessment before, during, and after the IV administration of vancomycin can help identify and prevent adverse effects, including nephrotoxicity, thrombophlebitis, extravasation, hypotension, hypersensitivity reaction, and red man syndrome. Test Id: 52003011 Question Id: 31303 (729561) A client with ulcerative colitis (UC) reports abdominal pain, 10 bloody stools per day, and decreased appetite. The client states, "What's the point of taking medication? It doesn't help anyway." Which nursing diagnoses (NDs) are appropriate to include in the client's plan of care? Select all that apply. Unordered Options Ordered Response 1. Acute pain 2. Altered nutritional status 3. Hopelessness 4. Noncompliance 5. Risk for deficient fluid volume Explanation UC is a chronic disease characterized by inflammation and ulcerations in the large intestines resulting in urgent, frequent, bloody diarrhea; abdominal pain; anorexia; and anemia. Appropriate NDs for a client with UC include: • Acute pain related to an intestinal inflammatory process as evidenced by the client's verbal reports of abdominal pain and self-focusing (Option 1) • Altered nutritional status related to decreased appetite and intestinal malabsorption (Option 2). • Hopelessness related to chronic illness and prolonged treatments with no positive results as evidenced by the client asking, "What's the point of taking medication? It doesn't help anyway" (Option 3). • Risk for deficient fluid volume related to active fluid losses secondary to frequent diarrhea; blood loss secondary to an inflamed bowel; and poor oral intake (Option 5). (Option 4) Noncompliance is failure to follow the prescribed treatment plan. This client has taken the medication but it did not help. Based on the assessment data, this is not an appropriate ND for the client at this time. Educational objective: UC is a chronic disease characterized by inflammation and ulcerations in the large intestines resulting in urgent, frequent, bloody diarrhea; abdominal pain; anorexia; and anemia. Acute pain, altered nutritional status, hopelessness, and risk for deficient fluid volume are appropriate NDs for a client with UC. Test Id: 52003011 Question Id: 31788 (729561) The health care provider suspects a rotator cuff injury in a client who is an avid tennis player. The nurse would most likely assess which of the following? Unordered Options Ordered Response 1. Complete stiffness of the shoulder joint 2. Paresthesia over the first 3½ fingers 3. Shoulder pain with arm abduction 4. Tenderness over the lateral epicondyle Explanation The rotator cuff is a group of 4 shoulder muscles and tendons that attach to the humeral head. It allows for rotation of the arm. A partial or full thickness rotator cuff tear can occur gradually over time as a result of aging, repetitive use, or an injury to the shoulder. It can also occur as a result of a sports injury involving repetitive overhead arm motion (eg, swimming, tennis, baseball, weight lifting). Characteristic symptoms of rotator cuff injury usually include shoulder pain and weakness. Severe pain when the arm is abducted between 60 and 120 degrees (painful arc) is characteristic (Option 3). (Option 1) Restriction of active and passive ranges of motion of the shoulder (complete stiffness) is seen with frozen shoulder. (Option 2) Pain and paresthesia over the first 3½ fingers suggest carpal tunnel syndrome. (Option 4) Tenderness over the lateral epicondyle is seen with tennis elbow. Educational objective: Rotator cuff injury involves a group of muscles and tendons in the shoulder. Characteristic symptoms of rotator cuff injury usually include shoulder pain and weakness. Severe pain when the arm is abducted between 60 and 120 degrees (painful arc) is characteristic. Test Id: 52003011 Question Id: 32079 (729561) A client sustained a concussion after falling off a ladder. What are essential instructions for the nurse to provide when the client is discharged from the hospital? Select all that apply. Unordered Options Ordered Response 1. Client should abstain from alcohol 2. Client should remain awake all night 3. Client should return if having difficulty walking 4. Responsible adult should be taught neurological examination 5. Responsible adult should stay with the client Explanation An essential aspect of discharging a client with a head injury is ensuring that a responsible adult will check on the client as the level of consciousness can change (Option 5). Brain edema or increased intracranial pressure (IICP) may not be evident immediately. The client should return to the emergency department or notify the primary care provider if any of the following signs/symptoms are present in the next 2-3 days: • Change in level of consciousness (eg, increased drowsiness, difficulty arousing, confusion) • Worsening headache or stiff neck, especially if unrelieved by over-the-counter analgesics • Visual changes (eg, blurring) • Motor problems (eg, difficulty walking, slurred speech) (Option 3) • Sensory disturbances • Seizures • Nausea/vomiting or bradycardia (indicates IICP) The client is also to abstain from alcohol, check before taking medications that can affect level of consciousness (eg, muscle relaxants, opioids), and avoid driving or operating heavy machinery (Option 1). (Option 2) It is not necessary to cause sleep deprivation by keeping the client awake. It is most important that the client can be easily aroused to an awake state. (Option 4) A neurological examination includes evaluation with the Glasgow Coma Scale, testing of pupils, and assessment of all 4 extremities for movement, strength, and sensation; this examination should be performed by a clinician. The responsible adult is taught the general indicative symptoms in the list above. Educational objective: A client sent home with a head injury requires the presence of a responsible adult. This person should observe for the signs/symptoms of increased intracranial pressure including change in level of consciousness, projectile vomiting, motor alteration (eg, ataxia), ipsilateral pupil dilation, and seizures. Test Id: 52003011 Question Id: 31183 (729561) The nurse in the postanesthesia care unit (PACU) is caring for an unresponsive client who just came from the operating room following surgery under general anesthetic for colorectal cancer. The nurse chooses what as the highest priority nursing diagnosis (ND)? Unordered Options Ordered Response 1. Acute pain 2. Impaired physical mobility 3. Ineffective airway clearance 4. Risk for fluid volume deficit Explanation Ineffective airway clearance, which is the inability to clear secretions or obstructions from the respiratory tract to maintain a clear airway, is the priority ND as it poses the greatest threat to survival. The most common causes of respiratory complications in the immediate postoperative period include the following: • Airway obstruction, which can be due to retained secretions or the tongue falling backward against the soft palate in sedated clients. Suctioning and an artificial oral airway can be used to prevent obstruction until the client becomes more responsive. • Hypoxemia, which can be due to atelectasis from increased retained secretions or hypoventilation, aspiration, or bronchospasm. Pulse oximetry and supplemental oxygen are used to maintain pulse oximeter readings >92%; placing the client in side-lying position and administrating antiemetic medications help to decrease aspiration. • Hypoventilation, which can be due to depression of the respiratory drive as a result of anesthesia, pain, and opioid analgesia. (Option 1) Most clients receive continual or bolus epidural or patient-controlled opioid analgesia to control postoperative pain. Pain assessment after surgery is a priority, and acute pain is an appropriate ND. However, it does not pose the greatest threat to survival and is not the priority diagnosis at this time. (Option 2) Ambulation and ability to reposition are important interventions to prevent multiple postoperative complications. However, impaired physical mobility is not the priority ND at this time. (Option 4) Risk for fluid volume deficit related to fluid losses during surgery is an appropriate diagnosis, but it does not pose the greatest threat to survival and is not the priority ND at this time. Educational objective: Acute pain, impaired physical mobility, ineffective airway clearance, and risk for fluid volume deficit are appropriate NDs in a client in the PACU who is immediately postoperative. The highest priority ND is the one that poses the greatest threat to survival based on the client's current health status. Test Id: 51996952 Question Id: 30894 (729561) A nurse on a medical surgical unit receives a report on multiple clients. Based on this report, which client should the nurse assess first? Unordered Options Ordered Response 1. A client who underwent a colon resection 3 hours ago and is bleeding 2. A client who was rescued from a burning building and shows evidence of smoke inhalation 3. A client with gastroenteritis who is throwing up large amounts of vomit 4. A client with peritonitis who has pain level of "8" on a scale from 1-10 You answered this question correctly. Explanation The ABC priority framework stands for airway, breathing, and circulation. This is the order in which clients should be assessed and treated. Smoke inhalation is the leading cause of death in burn clients as it causes thermal injury to the upper airways, chemical injury to the tracheobronchial tree, and carbon monoxide and/or cyanide poisoning. Clients should receive 100% oxygen to displace carbon monoxide and cyanide from hemoglobin. Intubation is indicated if there is evidence of upper airway edema with respiratory distress. An obstructed airway can lead to cardiac arrest if not treated immediately. (Option 1) According to the ABC priority framework, this client who is bleeding after a recent colon resection should be seen after the burn client has been treated. (Option 3) The client with gastroenteritis who is vomiting profusely should be assessed and given antiemetics and fluid resuscitation as ordered. However, this is not the priority in this situation. (Option 4) The client experiencing severe pain from peritonitis should certainly be assessed as soon as possible, but this is not the priority according to the ABC priority framework. Educational objective: The ABC priority framework is used to quickly evaluate the order in which clients should be assessed. This framework is appropriate in all settings, both emergency and non- emergency. Airway is the priority, followed by breathing, and finally circulation. Clients with acute asthma exacerbation, tension pneumothorax, anaphylaxis/angioedema, and flail chest need emergency evaluation. Test Id: 51996952 Question Id: 30891 (729561) The nurse determines that further teaching is needed if a client with constipation makes which statement? Select all that apply. Unordered Options Ordered Response 1. "I will go to the restroom when I have the urge to have a bowel movement." 2. "I will increase my exercise to at least 3 times a week." 3. "I will increase my intake of fruits and vegetables." 4. "I will increase tea or coffee consumption to stimulate the bowel." 5. "I will use a laxative every other day if needed." Explanation Constipation is a symptom of many different disease processes (eg, Parkinson's disease, diabetic neuropathy, depression), procedures (eg, abdominal surgery, bowel manipulation), and medications (eg, anticholinergics, diuretics, opioids, antacids, antihypertensives). Immobility, low-fiber diets, decreased fluid intake, and irregular bowel habits increase the likelihood of constipation. Teach the client and/or caregiver the following to prevent constipation: • Consume 20-30 g of fiber a day (unless contraindicated); fiber softens stool and increases bulk, stimulating defecation. High-fiber foods include fruits, vegetables, whole grains, nuts, seeds, and legumes (Option 3). • Drink 2-3 L of fluids a day (unless contraindicated); avoid caffeinated beverages (eg, coffee, tea, cola) that promote diuresis. • Exercise at least 3 times a week; movement stimulates peristalsis and defecation (Option 2) • Maintain a healthy bowel regimen – avoid delaying defecation when the urge is felt, defecate at the same time each day, and track bowel movements to identify if there is a change in bowel patterns (Option 1) • Avoid laxatives and enemas unless prescribed by a health care provider; overuse can cause dependency (Option 4) Consuming 8 glasses of water and fruit juices is recommended. Clients should avoid caffeinated beverages (eg, tea, cola, coffee) as they promote diuresis, which may lead to dehydration and worsening of constipation. (Option 5) Overuse of laxatives or enemas can lead to dependency and fluid and electrolyte imbalances. Educational objective: Healthy eating habits (consuming 20-30 g of fiber a day, drinking 2-3 L of fluids a day), exercise, and creating a bowel regimen (avoiding delay of defecation, defecating at the same time each day) are important practices that prevent constipation. Clients should avoid the regular use of laxatives and enemas as dependency may result. Test Id: 51996952 Question Id: 31398 (729561) The nurse receives report on the assigned team of clients on the oncology unit. All are receiving chemotherapy. Which client should the nurse check on first? Unordered Options Ordered Response 1. Alopecia and oral mucositis noted on assessment 2. Morning hemoglobin result is 8 g/dL (80 g/L) 3. New-onset back pain and weakness in legs 4. Persistent vomiting and potassium result is 3.4 mEq/L (3.4 mmol/L) Explanation A new-onset finding is more concerning than chronic or expected findings. There is a risk of spinal cord compression from a metastatic tumor in the epidural space. The classic symptoms are localized, persistent back pain; motor weakness; and sensory changes (eg, numbness, paresthesia). There can also be autonomic dysfunction, reflected by bowel or bladder dysfunction. Neurologic changes are a priority because the symptoms are subtle and time sensitive for permanent negative outcomes. Bone is a common site for metastasis due to its vascularity. This is the highest risk of the 4 options. (Option 1) Cells with rapid turnover take up the chemotherapy and are most affected. Commonly expected findings are alopecia (loss of hair) and mucositis/ulcers in the mouth/gastrointestinal system. Comfort care and teaching will be given for these signs, but they are not a priority over a neurologic abnormality. (Option 2) Chemotherapy suppresses bone marrow, commonly resulting in neutropenia (low neutrophil count), anemia (low hemoglobin), and thrombocytopenia (low platelets). This would be monitored but is not an emergency at this current level. (Option 4) Nausea and vomiting are expected side effects of chemotherapy and/or radiation treatment. Potassium losses occur with vomiting or diarrhea. However, this potassium result is only slightly lowered. It should be monitored but is not the priority compared to the new neurologic assessment. Educational objective: The client with cancer who experiences a new unexpected neurologic complication is a priority. Acute symptoms of spinal cord compression are a priority over stabilized anemia, minor hypokalemia, mucositis, or alopecia. Test Id: 51996952 Question Id: 30829 (729561) An experienced nurse is mentoring a new registered nurse (RN) on the telemetry unit. The new RN is measuring orthostatic blood pressure (BP) for a client. Which situation would warrant intervention by the experienced nurse? Unordered Options Ordered Response 1. Nurse has client lie supine for 5-10 minutes prior to starting procedure 2. Nurse interprets a decrease in systolic BP by 10 mm Hg as a normal finding 3. Nurse starts by measuring BP and heart rate (HR) with the client standing 4. Nurse takes BP and HR after standing at 1- and 3-minute intervals Explanation The experienced nurse should intervene if the new RN starts BP measurement with the client in the standing position. Orthostatic BP measurement may be done to detect volume depletion or postural hypotension caused by medications or autonomic dysfunction. Procedure for measurement of orthostatic BP 1. Have the client lie down for at least 5 minutes (Option 1) 2. Measure BP and HR 3. Have the client stand 4. Repeat BP and HR measurements after standing at 1- and 3-minute intervals (Option 4) A drop in systolic BP of ≥20 mm Hg or in diastolic BP of ≥10 mm Hg, or experiencing lightheadedness or dizziness is considered abnormal (Option 2). Educational objective: To measure orthostatic BP, the nurse should have the client lie supine for 5-10 minutes and then measure BP and HR. The nurse should then have the client stand for 1 minute, measure BP and HR, and repeat the measurements at 3 minutes. Findings are significant if the systolic BP drops ≥20 mm Hg or the diastolic BP drops ≥10 mm Hg. Test Id: 51996952 Question Id: 30153 (729561) The emergency department nurse cares for a client admitted with a diagnosis of hyperosmolar hyperglycemic state. The nurse understands which characteristics are commonly associated with this complication? Select all that apply. Unordered Options Ordered Response 1. Blood glucose level >600 mg/dL (33.3 mmol/L) 2. History of type 2 diabetes 3. Kussmaul respirations 4. Neurological manifestations 5. Abdominal pain Explanation Hyperosmolar hyperglycemic state is a serious complication usually associated with type 2 diabetes. With this condition, clients are able to produce enough insulin to prevent diabetic ketoacidosis but not enough to prevent extreme hyperglycemia, osmotic diuresis, and extracellular fluid deficit. Because some insulin is produced, blood glucose rises slowly and symptoms may not be recognized until hyperglycemia is extreme, often >600 mg/dL (33.3 mmol/L). This eventually causes neurological manifestations such as blurry vision, lethargy, obtundation, and progression to coma. Because some insulin is present, symptoms associated with ketones and acidosis, such as Kussmaul respirations (hyperventilation) and abdominal pain, are typically absent (Options 3 and 5). Educational objective: Hyperosmolar hyperglycemic state differs from diabetic ketoacidosis in that it is typically associated with type 2 diabetes mellitus. Because these clients produce some insulin, severe hyperglycemia happens more slowly and is often not noted until neurological manifestations occur. Test Id: 51996952 Question Id: 33540 (729561) The home care nurse visits the house of an elderly client. Which assessment finding requires immediate intervention? Unordered Options Ordered Response 1. The client cannot remember what was done yesterday 2. The client has a painful red area on the buttocks 3. The client has new dependent edema of the feet 4. The client has strong, foul smelling urine Explanation New onset of dependent edema of the feet could represent congestive heart failure. This is an urgent medical condition that needs prompt evaluation for characteristic signs (eg, weight gain, lung crackles) and treatment. (Option 1) Loss of short-term memory could be an early sign of dementia. It is important to assess clients' mental status to ensure safety in their homes. Further intervention is required, but this condition is not life-threatening. (Option 2) A painful red area on the buttocks represents the beginning stages of a pressure ulcer. Although not emergent, this does require further intervention. It is important to recognize pressure ulcers early and start treatment promptly before they progress to advanced stages. Advanced pressure ulcers are more difficult to treat and heal slower in the elderly. (Option 4) Strong, foul smelling urine is likely due to a urinary tract infection. This does require treatment to prevent further complications but is not a priority over suspected heart failure. Urinary tract infections can cause fever with confusion in the elderly. Educational objective: New onset of dependent edema in an elderly client could be due to heart failure; the client needs further assessment for characteristic signs such as lung crackles and increased body weight (fluid retention). Test Id: 51996952 Question Id: 32880 (729561) The nurse is preparing to administer a unit of packed red blood cells to a client whose hemoglobin is 7 g/dL (70 g/L). What tasks can the nurse delegate to unlicensed assistive personnel? Select all that apply. Unordered Options Ordered Response 1. Assist with checking identification of client and blood product 2. Measure vital signs at the end of the transfusion 3. Measure vital signs just after starting the transfusion 4. Measure vital signs prior to starting the transfusion 5. Obtain blood from the blood bank Explanation The registered nurse (RN) is responsible for most of the care rendered to a client during a blood transfusion as this is considered a high acuity procedure requiring a high level of nursing assessment and judgement. Based on the individual state practice act and institutional policy, the RN may have assistance from the licensed practical nurse (LPN) with checking blood products, verifying client identification, and monitoring the blood transfusion rate. Unlicensed assistive personnel (UAP) can obtain the blood product from the blood bank (Option 5) and take vital signs before the transfusion and after the first 15 minutes of infusion (Options 2 and 4). (Option 1) Only nurses are able to verify blood product and client identification for blood transfusion procedures. (Option 3) It is the responsibility of the RN to stay with the client during the first 15 minutes of the transfusion, monitor client response, and measure vital signs. A transfusion reaction is most likely to occur during this time. However, vital sign measurement may be delegated after the first 15 minutes have passed. Educational objective: After the registered nurse has initiated and monitored the client for the first 15 minutes of a blood transfusion, vital sign measurement may be delegated to unlicensed assistive personnel (UAP). UAP may also obtain the blood product from the blood bank. Test Id: 51996952 Question Id: 30022 (729561) The nurse is educating a client recently diagnosed with anaphylactic allergy to latex. Which statement made by the client indicates that the client understood the condition correctly? Unordered Options Ordered Response 1. "I do not need to worry about my allergy when I am outside of a health care environment." 2. "I just need to check labels to ensure products do not contain latex and I will be fine." 3. "I should always carry my Epi-pen in case I have difficulty breathing." 4. "I should take better care of myself and eat healthy foods like bananas and chestnuts." Explanation Anaphylactic shock is a medical emergency and the most severe form of an allergic reaction. Hives, itching, or a skin rash may or may not appear before rapid swelling of the mouth and throat (angioedema) makes breathing difficult or impossible within a span of minutes. Quick application of an epinephrine auto-injector (Epi-pen) into the thigh is the only acceptable option for treating anaphylactic shock. The intramuscular adrenaline injection immediately counteracts the life-threatening swelling, hypotension, and vasodilation that characterize anaphylaxis. Diphenhydramine (Benadryl) is also given to treat any associated rash or itching (hives, wheals, urticaria) but is not sufficient as a monotherapy. (Option 1) Latex products are everywhere. Clients and staff members should be educated and reminded about exposure to plastic products, condoms, and all other medical products containing latex. (Option 2) Numerous products may contain trace amounts of latex; this crucial information may be omitted on the labels. (Option 4) Bananas, avocados, chestnuts, and kiwifruit have been classified as having high-risk potential for cross-reaction allergy development. Clients should be advised to watch for potential allergic reactions due to a cross-allergen. Educational objective: Anaphylaxis is a medical emergency, and any client with a history of severe allergic reaction (sudden blotchy skin rash or swelling of the lips and mouth) should always carry an EpiPen. Epinephrine injection is the only option Test Id: 51996952 Question Id: 30621 (729561) A client arrives at the emergency department after being rescued from a burning building. Both arms and the entire chest are covered with dry, leathery, charred skin that does not blanch. What is the priority for the client's care? Unordered Options Ordered Response 1. Administration of IV lactated Ringer's 2. Antibiotic administration 3. Incision through the burned tissue (escharotomy) 4. IV administration of analgesics Explanation The initial management of burn injuries is identical to the management of all trauma clients - airway, breathing and circulation (ABCs) must always be secured first. All burn victims should be treated initially with high-flow oxygen via a nonrebreather mask, although caregivers should maintain a low threshold for intubation in any client with physical evidence of thermal damage to the upper airway. Following severe thermal burns, clients require significant volume replacement to compensate for fluid lost through their wounds and for a potential injury-related systemic inflammatory response causing increased capillary permeability with extravascular shift of fluids. Aggressive fluid replacement is essential if more than 15% total body surface area (TBSA) is involved. This client has 36% burn area based on the "Rule of Nines." Fluids used typically include lactated Ringer's. The amount of fluid required for the first 24 hours is calculated using the Parkland formula (4 mL/kg of body weight for each percent of TBSA burned). One-half of the total amount is infused in the first 8 hours, ¼ of the total in the 2nd 8 hours, and ¼ of the total in the 3rd 8 hours. (Option 2) Burn clients are at high risk for infection due to lack of skin integrity, especially by Pseudomonas aeruginosa, but treatment in the acute setting should focus on restoration of ABCs. (Option 3) Wound care is important but maintaining adequate circulation is a priority. This client has no evidence of a circumferential burn requiring an escharotomy. Even if this procedure is needed, fluids would be the first priority as the fluid loss/shift can start as early as 20 minutes after a burn and fluid resuscitation is life-saving. (Option 4) Full-thickness or 3rd-degree burns involve destruction of nerves, so there is no pain. There may be some other partial-thickness or superficial burns that do have severe pain. Under ABCD prioritization, pain is "D" and would be after circulation for fluids. Educational objective: The initial management of burn injuries is identical to the management of all trauma clients – ABCs must always be secured first. Fluids such as lactated Ringer's are typically used. Test Id: 51996952 Question Id: 30355 (729561) The nurse working in the intensive care unit hears an alarm coming from a client's room. On entering the room, the nurse sees the rhythm displayed in the exhibit on the monitor. The nurse recognizes it as which rhythm? Click on the exhibit button for additional information. Unordered Options Ordered Response 1. Asystole 2. Atrial fibrillation 3. Ventricular fibrillation (VF) 4. Ventricular tachycardia Explanation VF is characterized on the ECG by irregular waveforms of varying shapes and amplitudes. This represents the firing of multiple ectopic foci originating in the ventricle. Mechanically, the ventricle is quivering with no effective contraction or cardiac output. VF is considered a lethal dysrhythmia. It results in an unresponsive, pulseless, apneic state. If not treated rapidly, the client will not recover. VF commonly occurs in acute myocardial infarction and myocardial ischemia and in chronic heart diseases such as heart failure and cardiac myopathy. It may occur in cardiac pacing or catheterization procedures due to catheter stimulation of the ventricle. Treatment consists of rapid initiation of CPR, defibrillation, and the use of drug therapy (eg, epinephrine, vasopressin, amiodarone). (Option 1) Asystole is the total absence of ventricular electrical activity. (Option 2) Atrial fibrillation is characterized by total disorganization of atrial, not ventricular, activity. QRS complexes are usually normal in morphology. P waves are not seen. (Option 4) Ventricular tachycardia has a ventricular rate of 150-250/min and originates from foci firing repeatedly in the ventricle. Educational objective: The nurse should recognize VF, a potentially lethal dysrhythmia. The ECG shows irregular waveforms of varying shapes and amplitudes. The client is unresponsive, pulseless, and apneic. Rapid treatment should include CPR, defibrillation, and drug therapy (eg, epinephrine, vasopressin, amiodarone). Test Id: 51996952 Question Id: 30965 (729561) A grade-school client has type 1 diabetes controlled with glargine and aspart. The client becomes shaky, diaphoretic, and has slurred speech. What action should the school nurse take first? Unordered Options Ordered Response 1. Administer 1 tbs of honey 2. Give crackers with peanut butter 3. Inject 1 mg intramuscular glucagon 4. Provide an 8-oz glass of juice with 4 packs of sugar Explanation Classic signs and symptoms of hypoglycemia include shakiness, palpitations, nervousness, diaphoresis, anxiety, hunger, pallor, and changes in mental functioning (eg, difficulty speaking, visual disturbances, confusion). When hypoglycemic signs/symptoms are present, capillary blood glucose should be checked, if possible. Treatment is instituted if it is <70 mg/dL. If testing is not possible, the client should be treated. Appropriate treatment for suspected hypoglycemia is 15-20 g of quick-acting carbohydrate. Appropriate food choices would include 4-6 oz of regular soda, 8-10 LifeSavers candies, 1 tbs syrup or honey, 4 tsp jelly, 4-6 oz orange juice, 8 oz low-fat milk, or commercial dextrose products. The treatment is repeated in 15 minutes if there is no improvement. (Option 2) If the client's next meal is more than an hour away, a longer-acting carbohydrate and protein or fat is provided after the symptoms subside. This is an appropriate intake for that purpose; however, the hypoglycemia should be treated with a fast-acting carbohydrate first. (Option 3) If a client is unconscious, or symptoms/signs worsen, treatment should be subcutaneous or intramuscular injection of 1 mg glucagon. An alternate treatment is 25- 50 mL 50% glucose. (Option 4) Juice is a quick-acting carbohydrate but is overtreatment. Only 4-6 oz of juice should be used, without additional sugar. The risk of overtreatment is hyperglycemia and rebound hypoglycemia. Educational objective: Hypoglycemia should be treated with 15-20 g of quick-acting carbohydrate. Treatment should be repeated if there is no improvement in 15 minutes. Common initial treatment options include 1 tbs syrup or honey, 4 tsp jelly, 4-6 oz orange juice, or 8 oz low-fat milk. Avoid overtreatment to prevent rebound hypoglycemia. Test Id: 51996952 Question Id: 30118 (729561) A registered nurse is precepting a new nurse in the intensive care unit. The client is sedated with propofol, on a mechanical ventilator, and is receiving enteral feeding via nasogastric tube. The new nurse performs interventions to prevent aspiration. The preceptor should intervene if the new nurse performs which of the following actions? Unordered Options Ordered Response 1. Assesses gastric residual volumes every 4 hours 2. Measures the number of centimeters the feeding tube is secured at the nare every 4 hours 3. Requests that the physician change the client from continual to bolus feedings 4. Uses a sedation scale to titrate down the sedation (if possible) Explanation Critically ill clients are at increased risk for aspiration of oropharyngeal secretions and gastric content. It is common in clients who are intubated, sedated, on a mechanical ventilator, and receiving enteral feedings. The nurse must provide nursing interventions to prevent aspiration and monitor for its signs and symptoms. Clients are at increased risk when receiving bolus rather than continual enteral feedings. Bolus feedings should be avoided in critically ill clients, who are already at increased risk for aspiration. (Option 1) Assessing gastric residual volumes according to institution policy (at least every 4 hours) is standard for clients receiving continual enteral feedings. Increased volumes may indicate poor absorption and increase the risk of regurgitation and aspiration. (Option 2) Measuring the number of centimeters at the nare every 4 hours can help determine if the tube has moved, but it can increase aspiration risk. X-ray confirmation may be necessary if the tube has moved. (Option 4) A sedation scale such as the Ramsay Scale is used to assess level of sedation. It is preferable to keep the client minimally sedated (asleep but arousable). This helps decrease the risk of aspiration. Educational objective: Assessing gastric residual volumes and level of sedation at regular intervals, checking enteral feeding tube placement, and administering continual rather than bolus tube feeding are interventions that help prevent aspiration in critically ill high-risk clients. Copyright © UWorld. All rights reserved. Test Id: 51996952 Question Id: 33472 (729561) The nurse assesses and reviews the laboratory results for 4 clients. Which client's fever is of highest priority and should be reported to the health care provider immediately? Unordered Options Ordered Response Client newly diagnosed with Hodgkin lymphoma scheduled for 1. chemotherapy who has a fever of 100.9 F (38.3 C) and white blood cell count of 6,000/mm3 (6.0 × 109/L) Client with acute cholecystitis scheduled for laparoscopic surgery who has 2. a fever of 102 F (38.9 C) and white blood cell count of 13,000/mm3 (13.0 × 109/L) Client with Clostridium difficile infection receiving metronidazole who has 3. a fever of 101 F (38.3 C) and white blood cell count of 18,000/mm3 (18.0 × 109/L) 4. Client with colon cancer receiving chemotherapy who has a fever of 100.4 F (38 C) and white blood cell count of 1,500/mm3 (1.5 × 109/L) Explanation A common adverse effect of chemotherapy is bone marrow suppression (eg, anemia, leukopenia, thrombocytopenia) and immunosuppression. A decreased neutrophil (type of white blood cell) count, termed neutropenia, increases the client's susceptibility to infection. A fever can signal an infection and, in the presence of neutropenia (ie, neutropenic fever), can rapidly develop into life-threatening sepsis. Even a low-grade fever should be taken seriously in these clients. (Option 1) Hodgkin lymphoma is a malignant cancer of the lymphatic system. Expected early manifestations include painless enlarged lymph nodes, fatigue, fever, weight loss, and drenching night sweats. The client's white blood cell count is within normal limits (4,000-11,000 mm3 [4.0-11.0 × 109/L]). (Option 2) Acute cholecystitis involves inflammation of the gallbladder. Expected manifestations include right upper quadrant pain that can radiate to the right shoulder, nausea, vomiting, fever, and leukocytosis (white blood cells count >11,000/mm3 [11.0 × 109/L]). The client is scheduled for surgery and is likely on antibiotics. Even if the client is not on antibiotics, neutropenia is a priority over acute cholecystitis. (Option 3) Clostridium difficile is a toxin-producing bacterium that proliferates in the lower gastrointestinal tract. Expected manifestations include diarrhea, fever, and leukocytosis. First-line pharmacologic treatment includes metronidazole (Flagyl) and oral vancomycin. Educational objective: Common adverse effects of chemotherapy are bone marrow suppression (eg, anemia, leukopenia, thrombocytopenia) and immunosuppression. Even a low-grade fever should be taken seriously in clients who are immunosuppressed or have neutropenia. Test Id: 51996952 Question Id: 31511 (729561) Which nursing instruction is the highest priority when teaching a 38-year-old female client newly diagnosed with stress incontinence? Unordered Options Ordered Response 1. Coaching related to Kegel exercises 2. Importance of voiding every 2 hours 3. Minimizing caffeine and alcohol 4. Use of incontinence pads and pessary Explanation The nursing care plan for a client experiencing stress incontinence includes pelvic floor exercises, bladder training, incontinence products, and lifestyle modifications. • The highest priority for a client newly diagnosed with stress incontinence is preventing skin breakdown and urinary tract infections through bladder training. Teaching the client to empty the bladder every 2 hours when awake and every 4 hours at night reduces these risks (Option 2). • Pelvic floor exercises (eg, Kegel exercises), which strengthen the sphincter and structural supports of the bladder, are an essential part of the teaching plan but are not the priority for this client (Option 1). It will take approximately 6 weeks for pelvic floor muscle strength to improve. • Natural bladder irritants (eg, smoking, caffeine, alcohol) increase incontinence and should be eliminated but are not the priority in this client (Option 3). • Pessaries relieve minor pelvic organ prolapse and may be used in some clients when initial conservative measures fail. This client should receive initial instruction on the importance of emptying the bladder often (Option 4). Educational objective: Nursing interventions related to stress incontinence include bladder training (eg, voiding every 2 hours), pelvic floor exercises (eg, Kegel exercises), lifestyle modifications (weight loss, reduction of dietary bladder irritants, smoking cessation), and incontinence products. Test Id: 51996952 Question Id: 30888 (729561) Which of the following nursing interventions would the nurse implement when caring for a client newly diagnosed with acute viral hepatitis? Select all that apply. Unordered Options Ordered Response 1. Administer antiemetic medications as needed 2. Encourage a good breakfast and small, frequent meals 3. Promote rest periods alternating with periods of activity 4. Provide a diet high in protein and low in fat 5. Teach the client to abstain from alcohol Explanation Inflammation of the liver is present in acute viral hepatitis. Liver functions (eg, detoxifying the blood, manufacturing bile for lipid digestion) are disrupted, leading to signs and symptoms in various body systems. These include the digestive (eg, nausea, vomiting, anorexia, right upper-quadrant tenderness), urinary (eg, dark-colored urine), musculoskeletal (eg, fatigue, arthralgia, myalgia), and integumentary (eg, pruritus, jaundice) systems. Nursing interventions for the acute phase of hepatitis focus on resting the liver and providing nutrition for healing: 1. Rest o Alternate periods of rest and activity (Option 3) o Avoid alcohol and other drugs that increase liver metabolism (Option 5) o Medications (eg, appetite stimulants, antipruritics, analgesics) should be used cautiously to allow hepatocytes to heal. Antiemetics can be used to prevent nausea (Option 1). 2. Nutrition o Encourage small, frequent meals to decrease nausea. Anorexia is lowest in the morning; promote eating a larger breakfast (Option 2). o Provide oral care and avoid extremes in food temperature to increase appetite o Drink adequate amounts of fluid (2500-3000 mL/day) and encourage a diet high in carbohydrates and calories (Option 4) Clients with acute hepatitis should eat a diet high in calories and carbohydrates while decreasing fat and protein consumption. The liver produces bile, which aids in lipid digestion. A high-protein diet produces more ammonia and other toxic substances and the inflamed liver may not detoxify these well. Moderation of fat and protein intake allows the liver to rest. Educational objective: Acute viral hepatitis is treated with supportive measures, including rest (alternate activity and rest), avoiding alcohol and hepatotoxic medications, and adequate nutrition (increase calories and carbohydrates; eat small, frequent meals). Clients should reduce their consumption of fats and proteins, which increase liver metabolism. Test Id: 51996952 Question Id: 30337 (729561) A client with throat cancer is receiving radiation therapy to the head and neck. The best strategy for promoting oral intake is to encourage the client to do which of the following? Unordered Options Ordered Response 1. Chew sugar-free gum and sip liberal amounts of water with slices of lemon 2. Consume nutritional supplements 3. Perform oral hygiene before each meal 4. Use artificial saliva Explanation Two of the most common side effects of radiation therapy to the head and neck are xerostomia (dry mouth) and production of very thick mucus due to salivary gland hypofunction. Both conditions cause discomfort and can affect speech, taste, ability to swallow, and use of dentures. These adverse effects can significantly impact the client's nutritional status due to decreased oral intake. Chewing sugar-free gum will stimulate saliva production; sipping water frequently will help relieve the discomfort of xerostomia; and lemon juice (an acidic substance) will aid in thinning thick mucus. Other approaches for coping with xerostomia include sucking on sugar-free sour hard candy, and consuming flavored ices and ice pops, ice chips, and diluted fruit juices. Foods to avoid or limit include beverages containing caffeine and/or alcohol; sugary foods and beverages; and dry, crumbly foods. (Option 2) Nutritional supplements may be necessary to maintain the client's nutritional status, but they will not increase saliva production or help relieve the discomfort of thick secretions. (Option 3) Maintaining good oral hygiene is important for minimizing the risk of oral lesions and periodontal disease (caries). However, oral hygiene should be performed after meals and at bedtime. Oral hygiene protocols include the following measures: using a soft toothbrush, rinsing the mouth with a solution of salt and baking soda 4 times a day, using a fluoridated toothpaste, and flossing at least once daily. (Option 4) Artificial saliva can provide temporary relief for dry mouth; it does not promote saliva production. Sipping on water has been found to be just as effective as artificial saliva and is a less expensive choice if cost is a factor. Educational objective: Radiation therapy to the head and neck can cause xerostomia (dry mouth) and production of very thick mucus. Chewing sugar-free gum will stimulate saliva production, sipping water frequently will help relieve the discomfort of xerostomia, and lemon juice will aid in thinning thick mucus. Test Id: 51996952 Question Id: 30799 (729561) An adult client was severely burned in a warehouse accident. The client has second- degree burns on the right leg and right arm as well as the back. Using the rule of nines, the nurse estimates the percentage of the client's burns as which of these options? Unordered Options Ordered Response 1. 25% 2. 32% 3. 45% 4. 50% Explanation Using the rule of nines, the nurse would calculate the burn area as 45%. Right leg: 18% Right arm: 9% Back: 18% 18+18+9 = 45 Educational objective: The rule of nines is an estimated percentage of total body surface area burned in an adult. The head is 9%, anterior torso 18%, posterior torso 18%, each arm 9%, each leg 18%, and groin 1%. The rule of nines is often used at the initial evaluation and should be recalculated within the first 72 hours. Test Id: 51996952 Question Id: 30388 (729561) A client with dilated cardiomyopathy has the rhythm shown in the exhibit. Which action should the nurse take first? Unordered Options Ordered Response 1. Assess the client for a pulse 2. Assess the oxygen saturation 3. Initiate cardiopulmonary resuscitation (CPR) 4. Prepare to defibrillate the client Explanation Clients in ventricular tachycardia (VT) can be pulseless or have a pulse. Treatment is based on this important initial assessment. VT with a pulse should be further assessed for clinical stability or instability. Signs of instability include hypotension, altered mental status, signs of shock, chest pain, and acute heart failure. The unstable client in VT with a pulse is treated with synchronized cardioversion. The stable client in VT with a pulse is treated with antiarrhythmic medications (eg, amiodarone, procainamide, sotalol). (Option 2) Oxygen saturation should be assessed after the presence of a pulse has been established. (Options 3 and 4) CPR and defibrillation should be initiated only in a client who is pulseless. Educational objective: The client in VT must be assessed for the presence or absence of a pulse before further assessment or treatment is initiated. The unstable (hypotensive) client in VT with a pulse is treated with synchronized cardioversion. Copyright © UWorld. All rights reserved. [Show More]

Last updated: 1 year ago

Preview 1 out of 144 pages

Reviews( 0 )

Recommended For You

 *NURSING> UWorld > UWorld Maternity Nursing Test Questions and Answers (Verified 2021) (All)

preview
UWorld Maternity Nursing Test Questions and Answers (Verified 2021)

Maternity Nursing Test Id: 51538826 Question Id: 31554 (729561) A client at 34 weeks gestation has constipation. The client has been taking 325 mg ferrous sulfate tid for anemia since the la...

By A+ Solutions , Uploaded: Apr 18, 2021

$13.5

 *NURSING> UWorld > UWorld Pediatric Nursing Test Questions and Answers (Verified 2021). (All)

preview
UWorld Pediatric Nursing Test Questions and Answers (Verified 2021).

UWorld Pediatric Nursing Test Questions and Answers (Verified 2021).

By A+ Solutions , Uploaded: Apr 18, 2021

$15.5

 *NURSING> UWorld > UPDATED, UWorld Prioritization Nursing Test-Questions and Answers (Verified 2021) (All)

preview
UPDATED, UWorld Prioritization Nursing Test-Questions and Answers (Verified 2021)

UWorld Prioritization Nursing Test-Questions and Answers (Verified 2021)/UWorld Prioritization Nursing Test-Questions and Answers (Verified 2021)/UWorld Prioritization Nursing Test-Questions and Answe...

By A+ Solutions , Uploaded: Apr 18, 2021

$14.5

 *NURSING> UWorld > UWorld Maternity Nursing Test Questions and Answers (LATEST VERSION 2022/2023) (All)

preview
UWorld Maternity Nursing Test Questions and Answers (LATEST VERSION 2022/2023)

UWorld Maternity Nursing Test Questions and Answers (LATEST VERSION 2022/2023)///////UWorld Maternity Nursing Test Questions and Answers (LATEST VERSION 2022/2023)

By Rixx Dennis , Uploaded: Aug 26, 2022

$12.5

 *NURSING> UWorld > U World Immune final NCLEX Immune: Comprehensive Quick-Content to Guarantee Passing the UWorld Immune Final Exam. (All)

preview
U World Immune final NCLEX Immune: Comprehensive Quick-Content to Guarantee Passing the UWorld Immune Final Exam.

U World Immune final NCLEX Immune Methotrexate (Rheumatrex) classified as a folate antimetabolite, antineoplastic, immunosuppressant drug to treat various malignancies and as a nonbiologic disease-...

By QuizMaster , Uploaded: Sep 02, 2020

$9.5

 *NURSING> UWorld > U World Hematological Final. Nuclex Hematology. Comprehensive Quick-Content to Guarantee Passing the UWorld Hematological Final Exam. (All)

preview
U World Hematological Final. Nuclex Hematology. Comprehensive Quick-Content to Guarantee Passing the UWorld Hematological Final Exam.

lOMoARcPSD|784381 U World Hematological final nuclex hematology Cisplatin an antineoplastic medication that can cause renal toxicity. Warfarin (Coumadin) Anticoagulant given to clients with...

By QuizMaster , Uploaded: Sep 02, 2020

$9

 *NURSING> UWorld > U World Maternal Health Nursing- Med Surg: Exam Study Guide with Last minute Information you NEED to pass the UWorld Maternal Health Exam. (All)

preview
U World Maternal Health Nursing- Med Surg: Exam Study Guide with Last minute Information you NEED to pass the UWorld Maternal Health Exam.

lOMoARcPSD|784381 U World Maternal Health Nursing- Med Surg Ectopic pregnancy Symptoms include lower abdominal and pelvic pain; amenorrhea, possibly followed by vaginal Important preconception...

By QuizMaster , Uploaded: Sep 02, 2020

$9.5

 *NURSING> UWorld > UWorld Neuro Nursing- Med Surg. This exam Study Guide Contains the Last minute Information you NEED to pass the UWorld Neuro Exam. An easy to read layout. (All)

preview
UWorld Neuro Nursing- Med Surg. This exam Study Guide Contains the Last minute Information you NEED to pass the UWorld Neuro Exam. An easy to read layout.

lOMoARcPSD|784381 U World Neuro Nursing- Med Surg parietal lobe receives sensory input frontal lobe Controls higher-order processing, such as executive function and personality.. tem...

By QuizMaster , Uploaded: Sep 02, 2020

$11

 *NURSING> UWorld > SKINNY Reasoning Urinary Tract Infection/Urosepsis Answer key;Jean Kelly, 82 years old (All)

preview
SKINNY Reasoning Urinary Tract Infection/Urosepsis Answer key;Jean Kelly, 82 years old

SKINNY Reasoning Urinary Tract Infection/Urosepsis Answer key;Jean Kelly, 82 years old

By Aimhigh , Uploaded: Jul 22, 2022

$9

 *NURSING> UWorld > Unfolding clinical Reasoning Case Study;Appendicitis/Appendectomy RAPID Reasoning Suggested Answer Guidelines,John Washington, 14 years old (All)

preview
Unfolding clinical Reasoning Case Study;Appendicitis/Appendectomy RAPID Reasoning Suggested Answer Guidelines,John Washington, 14 years old

Unfolding clinical Reasoning Case Study;Appendicitis/Appendectomy RAPID Reasoning Suggested Answer Guidelines,John Washington, 14 years old

By Aimhigh , Uploaded: Jul 22, 2022

$9.5

$15.50

Add to cart

Instant download

Can't find what you want? Try our AI powered Search

OR

GET ASSIGNMENT HELP
216
0

Document information


Connected school, study & course



About the document


Uploaded On

Apr 18, 2021

Number of pages

144

Written in

Seller


seller-icon
A+ Solutions

Member since 3 years

163 Documents Sold


Additional information

This document has been written for:

Uploaded

Apr 18, 2021

Downloads

 0

Views

 216

Document Keyword Tags

THE BEST STUDY GUIDES

Avoid resits and achieve higher grades with the best study guides, textbook notes, and class notes written by your fellow students

custom preview

Avoid examination resits

Your fellow students know the appropriate material to use to deliver high quality content. With this great service and assistance from fellow students, you can become well prepared and avoid having to resits exams.

custom preview

Get the best grades

Your fellow student knows the best materials to research on and use. This guarantee you the best grades in your examination. Your fellow students use high quality materials, textbooks and notes to ensure high quality

custom preview

Earn from your notes

Get paid by selling your notes and study materials to other students. Earn alot of cash and help other students in study by providing them with appropriate and high quality study materials.


$15.50

WHAT STUDENTS SAY ABOUT US


What is Browsegrades

In Browsegrades, a student can earn by offering help to other student. Students can help other students with materials by upploading their notes and earn money.

We are here to help

We're available through e-mail, Twitter, Facebook, and live chat.
 FAQ
 Questions? Leave a message!

Follow us on
 Twitter

Copyright © Browsegrades · High quality services·